Programs & Examples On #Taglib

TagLibs is a shortened form of "Tag Libraries". Tag Libraries are libraries that include "tags". Tags are usually used to address presentation concerns and encapsulate view-specific logic.

Eclipse "cannot find the tag library descriptor" for custom tags (not JSTL!)

For me, this error occurs whenever I try to use a new version of eclipse. Apparently, the new eclipse resets the M2_REPO variable and I get all the tag library error in the Marker view (sometimes with ejb validation errors).

After updating M2_REPO variable to point to actual maven repository location, it takes 2-3 Project -> Clean iterations to get everything working.

And sometimes, there are some xml validation errors(ejb) along with this tag library errors. Manually updating the corresponding XML file, initiates a *.xsd file lookup and the xml validations errors are resolved. Post this, the tag library errors also vanish.

Running CMake on Windows

There is a vcvars32.bat in your Visual Studio installation directory. You can add call cmd.exe at the end of that batch program and launch it. From that shell you can use CMake or cmake-gui and cl.exe would be known to CMake.

Sorting dictionary keys in python

[v[0] for v in sorted(foo.items(), key=lambda(k,v): (v,k))]

Does C# have an equivalent to JavaScript's encodeURIComponent()?

I tried to do full compatible analog of javascript's encodeURIComponent for c# and after my 4 hour experiments I found this

c# CODE:

string a = "!@#$%^&*()_+ some text here ??? ??????? ????";
a = System.Web.HttpUtility.UrlEncode(a);
a = a.Replace("+", "%20");

the result is: !%40%23%24%25%5e%26*()_%2b%20some%20text%20here%20%d0%b0%d0%bb%d0%b8%20%d0%bc%d0%b0%d0%bc%d0%b5%d0%b4%d0%be%d0%b2%20%d0%b1%d0%b0%d0%ba%d1%83

After you decode It with Javascript's decodeURLComponent();

you will get this: !@#$%^&*()_+ some text here ??? ??????? ????

Thank You for attention

Show/hide widgets in Flutter programmatically

One solution is to set tis widget color property to Colors.transparent. For instance:

IconButton(
    icon: Image.asset("myImage.png",
        color: Colors.transparent,
    ),
    onPressed: () {},
),

"The import org.springframework cannot be resolved."

My direct solution for this issue : right click the project --> Maven ---> Add Dependency == then choose the name or parent name of missing dependency

enter image description here

Sending an HTTP POST request on iOS

Using Swift 3 or 4 you can access these http request for sever communication.

// For POST data to request

 func postAction()  {
//declare parameter as a dictionary which contains string as key and value combination. considering inputs are valid
let parameters = ["id": 13, "name": "jack"] as [String : Any]
//create the url with URL
let url = URL(string: "www.requestURL.php")! //change the url
//create the session object
let session = URLSession.shared
//now create the URLRequest object using the url object
var request = URLRequest(url: url)
request.httpMethod = "POST" //set http method as POST
do {
    request.httpBody = try JSONSerialization.data(withJSONObject: parameters, options: .prettyPrinted) // pass dictionary to nsdata object and set it as request body
} catch let error {
    print(error.localizedDescription)
}
request.addValue("application/json", forHTTPHeaderField: "Content-Type")
request.addValue("application/json", forHTTPHeaderField: "Accept")
//create dataTask using the session object to send data to the server
let task = session.dataTask(with: request as URLRequest, completionHandler: { data, response, error in
    guard error == nil else {
        return
    }
    guard let data = data else {
        return
    }
    do {
        //create json object from data
        if let json = try JSONSerialization.jsonObject(with: data, options: .mutableContainers) as? [String: Any] {
            print(json)
            // handle json...
        }
    } catch let error {
        print(error.localizedDescription)
    }
})
task.resume() }

// For get the data from request

func GetRequest()  {
    let urlString = URL(string: "http://www.requestURL.php") //change the url

    if let url = urlString {
        let task = URLSession.shared.dataTask(with: url) { (data, response, error) in
            if error != nil {
                print(error ?? "")
            } else {
                if let responceData = data {
                    print(responceData) //JSONSerialization
                    do {
                        //create json object from data
                        if let json = try JSONSerialization.jsonObject(with:responceData, options: .mutableContainers) as? [String: Any] {
                            print(json)
                            // handle json...
                        }
                    } catch let error {
                        print(error.localizedDescription)
                    }
                }
            }
        }
        task.resume()
    }
}

// For get the download content like image or video from request

func downloadTask()  {
    // Create destination URL
    let documentsUrl:URL =  FileManager.default.urls(for: .documentDirectory, in: .userDomainMask).first as URL!
    let destinationFileUrl = documentsUrl.appendingPathComponent("downloadedFile.jpg")
    //Create URL to the source file you want to download
    let fileURL = URL(string: "http://placehold.it/120x120&text=image1")
    let sessionConfig = URLSessionConfiguration.default
    let session = URLSession(configuration: sessionConfig)
    let request = URLRequest(url:fileURL!)

    let task = session.downloadTask(with: request) { (tempLocalUrl, response, error) in
        if let tempLocalUrl = tempLocalUrl, error == nil {
            // Success
            if let statusCode = (response as? HTTPURLResponse)?.statusCode {
                print("Successfully downloaded. Status code: \(statusCode)")
            }

            do {
                try FileManager.default.copyItem(at: tempLocalUrl, to: destinationFileUrl)
            } catch (let writeError) {
                print("Error creating a file \(destinationFileUrl) : \(writeError)")
            }

        } else {
            print("Error took place while downloading a file. Error description: %@", error?.localizedDescription ?? "");
        }
    }
    task.resume()

}

Uncaught SyntaxError: Unexpected token with JSON.parse

products = [{"name":"Pizza","price":"10","quantity":"7"}, {"name":"Cerveja","price":"12","quantity":"5"}, {"name":"Hamburguer","price":"10","quantity":"2"}, {"name":"Fraldas","price":"6","quantity":"2"}];

change to

products = '[{"name":"Pizza","price":"10","quantity":"7"}, {"name":"Cerveja","price":"12","quantity":"5"}, {"name":"Hamburguer","price":"10","quantity":"2"}, {"name":"Fraldas","price":"6","quantity":"2"}]';

Copying files from server to local computer using SSH

Make sure the scp command is available on both sides - both on the client and on the server.

BOTH Server and Client, otherwise you will encounter this kind of (weird)error message on your client: scp: command not found or something similar even though though you have it all configured locally.

Selected value for JSP drop down using JSTL

You can try one even more simple:

<option value="1" ${item.quantity == 1 ? "selected" : ""}>1</option>

Use cases for the 'setdefault' dict method

Here are some examples of setdefault to show its usefulness:

"""
d = {}
# To add a key->value pair, do the following:
d.setdefault(key, []).append(value)

# To retrieve a list of the values for a key
list_of_values = d[key]

# To remove a key->value pair is still easy, if
# you don't mind leaving empty lists behind when
# the last value for a given key is removed:
d[key].remove(value)

# Despite the empty lists, it's still possible to 
# test for the existance of values easily:
if d.has_key(key) and d[key]:
    pass # d has some values for key

# Note: Each value can exist multiple times!
"""
e = {}
print e
e.setdefault('Cars', []).append('Toyota')
print e
e.setdefault('Motorcycles', []).append('Yamaha')
print e
e.setdefault('Airplanes', []).append('Boeing')
print e
e.setdefault('Cars', []).append('Honda')
print e
e.setdefault('Cars', []).append('BMW')
print e
e.setdefault('Cars', []).append('Toyota')
print e

# NOTE: now e['Cars'] == ['Toyota', 'Honda', 'BMW', 'Toyota']
e['Cars'].remove('Toyota')
print e
# NOTE: it's still true that ('Toyota' in e['Cars'])

Insert using LEFT JOIN and INNER JOIN

INSERT INTO Test([col1],[col2]) (
    SELECT 
        a.Name AS [col1],
        b.sub AS [col2] 
    FROM IdTable b 
    INNER JOIN Nametable a ON b.no = a.no
)

Is there a function to round a float in C or do I need to write my own?

There is a round() function, also fround(), which will round to the nearest integer expressed as a double. But that is not what you want.

I had the same problem and wrote this:

#include <math.h>

   double db_round(double value, int nsig)
/* ===============
**
** Rounds double <value> to <nsig> significant figures.  Always rounds
** away from zero, so -2.6 to 1 sig fig will become -3.0.
**
** <nsig> should be in the range 1 - 15
*/

{
    double     a, b;
    long long  i;
    int        neg = 0;


    if(!value) return value;

    if(value < 0.0)
    {
        value = -value;
        neg = 1;
    }

    i = nsig - log10(value);

    if(i) a = pow(10.0, (double)i);
    else  a = 1.0;

    b = value * a;
    i = b + 0.5;
    value = i / a;

    return neg ? -value : value;
} 

Dynamically create and submit form

Assuming you want create a form with some parameters and make a POST call

var param1 = 10;

$('<form action="./your_target.html" method="POST">' +
'<input type="hidden" name="param" value="' + param + '" />' +
'</form>').appendTo('body').submit();

You could also do it all on one line if you so wish :-)

jQuery add required to input fields

Using .attr method

.attr(attribute,value); // syntax

.attr("required", true);
// required="required"

.attr("required", false);
// 

Using .prop

.prop(property,value) // syntax

.prop("required", true);
// required=""

.prop("required", false);
//

Read more from here

https://stackoverflow.com/a/5876747/5413283

How to verify CuDNN installation?

Run ./mnistCUDNN in /usr/src/cudnn_samples_v7/mnistCUDNN

Here is an example:

cudnnGetVersion() : 7005 , CUDNN_VERSION from cudnn.h : 7005 (7.0.5)
Host compiler version : GCC 5.4.0
There are 1 CUDA capable devices on your machine :
device 0 : sms 30  Capabilities 6.1, SmClock 1645.0 Mhz, MemSize (Mb) 24446, MemClock 4513.0 Mhz, Ecc=0,    boardGroupID=0
Using device 0

List Highest Correlation Pairs from a Large Correlation Matrix in Pandas?

I liked Addison Klinke's post the most, as being the simplest, but used Wojciech Moszczynsk’s suggestion for filtering and charting, but extended the filter to avoid absolute values, so given a large correlation matrix, filter it, chart it, and then flatten it:

Created, Filtered and Charted

dfCorr = df.corr()
filteredDf = dfCorr[((dfCorr >= .5) | (dfCorr <= -.5)) & (dfCorr !=1.000)]
plt.figure(figsize=(30,10))
sn.heatmap(filteredDf, annot=True, cmap="Reds")
plt.show()

filtered heat map

Function

In the end, I created a small function to create the correlation matrix, filter it, and then flatten it. As an idea, it could easily be extended, e.g., asymmetric upper and lower bounds, etc.

def corrFilter(x: pd.DataFrame, bound: float):
    xCorr = x.corr()
    xFiltered = xCorr[((xCorr >= bound) | (xCorr <= -bound)) & (xCorr !=1.000)]
    xFlattened = xFiltered.unstack().sort_values().drop_duplicates()
    return xFlattened

corrFilter(df, .7)

enter image description here

Command to get time in milliseconds

  • date +"%T.%N" returns the current time with nanoseconds.

    06:46:41.431857000
    
  • date +"%T.%6N" returns the current time with nanoseconds rounded to the first 6 digits, which is microseconds.

    06:47:07.183172
    
  • date +"%T.%3N" returns the current time with nanoseconds rounded to the first 3 digits, which is milliseconds.

    06:47:42.773
    

In general, every field of the date command's format can be given an optional field width.

undefined reference to 'vtable for class' constructor

You're declaring a virtual function and not defining it:

virtual void calculateCredits();

Either define it or declare it as:

virtual void calculateCredits() = 0;

Or simply:

virtual void calculateCredits() { };

Read more about vftable: http://en.wikipedia.org/wiki/Virtual_method_table

Could not find server 'server name' in sys.servers. SQL Server 2014

At first check out that your linked server is in the list by this query

select name from sys.servers

If it not exists then try to add to the linked server

EXEC sp_addlinkedserver @server = 'SERVER_NAME' --or may be server ip address

After that login to that linked server by

EXEC sp_addlinkedsrvlogin 'SERVER_NAME'
                         ,'false'
                         ,NULL
                         ,'USER_NAME'
                         ,'PASSWORD'

Then you can do whatever you want ,treat it like your local server

exec [SERVER_NAME].[DATABASE_NAME].dbo.SP_NAME @sample_parameter

Finally you can drop that server from linked server list by

sp_dropserver 'SERVER_NAME', 'droplogins'

If it will help you then please upvote.

What is your single most favorite command-line trick using Bash?

This prevents less (less is more) from clearing the screen at the end of a file:

export LESS="-X"

How do I configure Maven for offline development?


(source: jfrog.com)

or

Just use Maven repository servers like Sonatype Nexus http://www.sonatype.org/nexus/ or JFrog Artifactory https://www.jfrog.com/artifactory/.

After one developer builds a project, build by next developers or Jenkins CI will not require Internet access.

Maven repository server also can have proxies configured to access Maven Central (or more needed public repositories), and they can have cynch'ed list of artifacts in remote repositories.

How to show a confirm message before delete?

The onclick handler should return false after the function call. For eg.

onclick="ConfirmDelete(); return false;">

Installing Google Protocol Buffers on mac

For some reason I need to use protobuf 2.4.1 in my project on OS X El Capitan. However homebrew has removed protobuf241 from its formula. I install it according @kksensei's answer manually and have to fix some error during the process.

During the make process, I get 3 error like following:

_x000D_
_x000D_
google/protobuf/message.cc:130:60: error: implicit instantiation of undefined template 'std::__1::basic_istream<char, std::__1::char_traits<char> >'_x000D_
_x000D_
  return ParseFromZeroCopyStream(&zero_copy_input) && input->eof();_x000D_
_x000D_
                                                           ^_x000D_
_x000D_
/Applications/Xcode.app/Contents/Developer/Toolchains/XcodeDefault.xctoolchain/usr/bin/../lib/c++/v1/iosfwd:108:28: note: template is declared here_x000D_
_x000D_
    class _LIBCPP_TYPE_VIS basic_istream;_x000D_
_x000D_
                           ^_x000D_
_x000D_
google/protobuf/message.cc:135:67: error: implicit instantiation of undefined template 'std::__1::basic_istream<char, std::__1::char_traits<char> >'_x000D_
_x000D_
  return ParsePartialFromZeroCopyStream(&zero_copy_input) && input->eof();_x000D_
_x000D_
                                                                  ^_x000D_
_x000D_
/Applications/Xcode.app/Contents/Developer/Toolchains/XcodeDefault.xctoolchain/usr/bin/../lib/c++/v1/iosfwd:108:28: note: template is declared here_x000D_
_x000D_
    class _LIBCPP_TYPE_VIS basic_istream;_x000D_
_x000D_
                           ^_x000D_
_x000D_
google/protobuf/message.cc:175:16: error: implicit instantiation of undefined template 'std::__1::basic_ostream<char, std::__1::char_traits<char> >'_x000D_
_x000D_
  return output->good();_x000D_
_x000D_
               ^_x000D_
_x000D_
/Applications/Xcode.app/Contents/Developer/Toolchains/XcodeDefault.xctoolchain/usr/bin/../lib/c++/v1/iosfwd:110:28: note: template is declared here_x000D_
_x000D_
    class _LIBCPP_TYPE_VIS basic_ostream;_x000D_
_x000D_
                           ^
_x000D_
_x000D_
_x000D_

(Sorry, I dont know how to attach code when the code contains '`' )

If you get the same error, please edit src/google/protobuf/message.cc, add #include <istream> at the top of the file and do $ make again and should get no errors. After that do $ sudo make install. When install finished $protoc --version should display the correct result.

log4net vs. Nlog

Shameless plug for an open source project I run, but given the lively discussion about which .NET logging framework is more active I thought I'd post an obligatory link to Serilog.

To use within an application, Serilog is similar to (and draws heavily on) log4net. Unlike other .NET logging options, however, Serilog is about preserving the structure of log events for offline analysis. When you write:

Log.Information("The answer is {Answer}", 42);

Most logging libraries immediately render the message into a string. Serilog can do that too, but it preserves the { Answer: 42 } property so that later on, using one of a number of NoSQL data stores, you can properly query events based on the value of Answer.

We're close to a 1.0 and support all of the modern (.NET 4.5, Windows Store and Windows Phone 8) platforms.

What is the proper way to format a multi-line dict in Python?

First of all, like Steven Rumbalski said, "PEP8 doesn't address this question", so it is a matter of personal preference.

I would use a similar but not identical format as your format 3. Here is mine, and why.

my_dictionary = { # Don't think dict(...) notation has more readability
    "key1": 1, # Indent by one press of TAB (i.e. 4 spaces)
    "key2": 2, # Same indentation scale as above
    "key3": 3, # Keep this final comma, so that future addition won't show up as 2-lines change in code diff
    } # My favorite: SAME indentation AS ABOVE, to emphasize this bracket is still part of the above code block!
the_next_line_of_code() # Otherwise the previous line would look like the begin of this part of code

bad_example = {
               "foo": "bar", # Don't do this. Unnecessary indentation wastes screen space
               "hello": "world" # Don't do this. Omitting the comma is not good.
} # You see? This line visually "joins" the next line when in a glance
the_next_line_of_code()

btw_this_is_a_function_with_long_name_or_with_lots_of_parameters(
    foo='hello world',  # So I put one parameter per line
    bar=123,  # And yeah, this extra comma here is harmless too;
              # I bet not many people knew/tried this.
              # Oh did I just show you how to write
              # multiple-line inline comment here?
              # Basically, same indentation forms a natural paragraph.
    ) # Indentation here. Same idea as the long dict case.
the_next_line_of_code()

# By the way, now you see how I prefer inline comment to document the very line.
# I think this inline style is more compact.
# Otherwise you will need extra blank line to split the comment and its code from others.

some_normal_code()

# hi this function is blah blah
some_code_need_extra_explanation()

some_normal_code()

Concatenating two std::vectors

A general performance boost for concatenate is to check the size of the vectors. And merge/insert the smaller one with the larger one.

//vector<int> v1,v2;
if(v1.size()>v2.size()) {
    v1.insert(v1.end(),v2.begin(),v2.end());
} else {
    v2.insert(v2.end(),v1.begin(),v1.end());
}

How can I capitalize the first letter of each word in a string using JavaScript?

If you can use a third-party library then Lodash has a helper function for you.

https://lodash.com/docs/4.17.3#startCase

_x000D_
_x000D_
_.startCase('foo bar');
// => 'Foo Bar'

_.startCase('--foo-bar--');
// => 'Foo Bar'

_.startCase('fooBar');
// => 'Foo Bar'

_.startCase('__FOO_BAR__');
// => 'FOO BAR'
_x000D_
<script src="https://cdn.jsdelivr.net/lodash/4.17.3/lodash.min.js"></script>
_x000D_
_x000D_
_x000D_

List all indexes on ElasticSearch server?

If you're working in scala, a way to do this and use Future's is to create a RequestExecutor, then use the IndicesStatsRequestBuilder and the administrative client to submit your request.

import org.elasticsearch.action.{ ActionRequestBuilder, ActionListener, ActionResponse }
import scala.concurrent.{ Future, Promise, blocking }

/** Convenice wrapper for creating RequestExecutors */
object RequestExecutor {
    def apply[T <: ActionResponse](): RequestExecutor[T] = {
        new RequestExecutor[T]
    }
}

/** Wrapper to convert an ActionResponse into a scala Future
 *
 *  @see http://chris-zen.github.io/software/2015/05/10/elasticsearch-with-scala-and-akka.html
 */
class RequestExecutor[T <: ActionResponse] extends ActionListener[T] {
    private val promise = Promise[T]()

    def onResponse(response: T) {
        promise.success(response)
    }

    def onFailure(e: Throwable) {
        promise.failure(e)
    }

    def execute[RB <: ActionRequestBuilder[_, T, _, _]](request: RB): Future[T] = {
        blocking {
            request.execute(this)
            promise.future
        }
    }
}

The executor is lifted from this blog post which is definitely a good read if you're trying to query ES programmatically and not through curl. One you have this you can create a list of all indexes pretty easily like so:

def totalCountsByIndexName(): Future[List[(String, Long)]] = {
    import scala.collection.JavaConverters._
    val statsRequestBuider = new IndicesStatsRequestBuilder(client.admin().indices())
    val futureStatResponse = RequestExecutor[IndicesStatsResponse].execute(statsRequestBuider)
    futureStatResponse.map { indicesStatsResponse =>
        indicesStatsResponse.getIndices().asScala.map {
            case (k, indexStats) => {
                val indexName = indexStats.getIndex()
                val totalCount = indexStats.getTotal().getDocs().getCount()
                    (indexName, totalCount)
                }
        }.toList
    }
}

client is an instance of Client which can be a node or a transport client, whichever suits your needs. You'll also need to have an implicit ExecutionContext in scope for this request. If you try to compile this code without it then you'll get a warning from the scala compiler on how to get that if you don't have one imported already.

I needed the document count, but if you really only need the names of the indices you can pull them from the keys of the map instead of from the IndexStats:

indicesStatsResponse.getIndices().keySet()

This question shows up when you're searching for how to do this even if you're trying to do this programmatically, so I hope this helps anyone looking to do this in scala/java. Otherwise, curl users can just do as the top answer says and use

curl http://localhost:9200/_aliases

Core dump file analysis

Steps to debug coredump using GDB:

Some generic help:

gdb start GDB, with no debugging les

gdb program begin debugging program

gdb program core debug coredump core produced by program

gdb --help describe command line options

  1. First of all, find the directory where the corefile is generated.

  2. Then use ls -ltr command in the directory to find the latest generated corefile.

  3. To load the corefile use

    gdb binary path of corefile
    

    This will load the corefile.

  4. Then you can get the information using the bt command.

    For a detailed backtrace use bt full.

  5. To print the variables, use print variable-name or p variable-name

  6. To get any help on GDB, use the help option or use apropos search-topic

  7. Use frame frame-number to go to the desired frame number.

  8. Use up n and down n commands to select frame n frames up and select frame n frames down respectively.

  9. To stop GDB, use quit or q.

Python, compute list difference

A = [1,2,3,4]
B = [2,5]

#A - B
x = list(set(A) - set(B))
#B - A 
y = list(set(B) - set(A))

print x
print y 

What is the purpose of a plus symbol before a variable?

As explained in other answers it converts the variable to a number. Specially useful when d can be either a number or a string that evaluates to a number.

Example (using the addMonths function in the question):

addMonths(34,1,true);
addMonths("34",1,true);

then the +d will evaluate to a number in all cases. Thus avoiding the need to check for the type and take different code paths depending on whether d is a number, a function or a string that can be converted to a number.

What is PAGEIOLATCH_SH wait type in SQL Server?

PAGEIOLATCH_SH wait type usually comes up as the result of fragmented or unoptimized index.

Often reasons for excessive PAGEIOLATCH_SH wait type are:

  • I/O subsystem has a problem or is misconfigured
  • Overloaded I/O subsystem by other processes that are producing the high I/O activity
  • Bad index management
  • Logical or physical drive misconception
  • Network issues/latency
  • Memory pressure
  • Synchronous Mirroring and AlwaysOn AG

In order to try and resolve having high PAGEIOLATCH_SH wait type, you can check:

  • SQL Server, queries and indexes, as very often this could be found as a root cause of the excessive PAGEIOLATCH_SH wait types
  • For memory pressure before jumping into any I/O subsystem troubleshooting

Always keep in mind that in case of high safety Mirroring or synchronous-commit availability in AlwaysOn AG, increased/excessive PAGEIOLATCH_SH can be expected.

You can find more details about this topic in the article Handling excessive SQL Server PAGEIOLATCH_SH wait types

Maximum length for MD5 input/output

A 128-bit MD5 hash is represented as a sequence of 32 hexadecimal digits.

How to solve java.lang.NullPointerException error?

A NullPointerException means that one of the variables you are passing is null, but the code tries to use it like it is not.

For example, If I do this:

Integer myInteger = null;
int n = myInteger.intValue();

The code tries to grab the intValue of myInteger, but since it is null, it does not have one: a null pointer exception happens.

What this means is that your getTask method is expecting something that is not a null, but you are passing a null. Figure out what getTask needs and pass what it wants!

Using sed to split a string with a delimiter

Using simply :

$ tr ':' $'\n' <<< string1:string2:string3:string4:string5
string1
string2
string3
string4
string5

If you really need :

$ sed 's/:/\n/g' <<< string1:string2:string3:string4:string5
string1
string2
string3
string4
string5

How to check the maximum number of allowed connections to an Oracle database?

select count(*),sum(decode(status, 'ACTIVE',1,0)) from v$session where type= 'USER'

How do I write a Python dictionary to a csv file?

Your code was very close to working.

Try using a regular csv.writer rather than a DictWriter. The latter is mainly used for writing a list of dictionaries.

Here's some code that writes each key/value pair on a separate row:

import csv

somedict = dict(raymond='red', rachel='blue', matthew='green')
with open('mycsvfile.csv','wb') as f:
    w = csv.writer(f)
    w.writerows(somedict.items())

If instead you want all the keys on one row and all the values on the next, that is also easy:

with open('mycsvfile.csv','wb') as f:
    w = csv.writer(f)
    w.writerow(somedict.keys())
    w.writerow(somedict.values())

Pro tip: When developing code like this, set the writer to w = csv.writer(sys.stderr) so you can more easily see what is being generated. When the logic is perfected, switch back to w = csv.writer(f).

How to change the display name for LabelFor in razor in mvc3?

Decorate the model property with the DisplayName attribute.

How to use boolean 'and' in Python

In python, use and instead of && like this:

#!/usr/bin/python
foo = True;
bar = True;
if foo and bar:
    print "both are true";

This prints:

both are true

Finding the direction of scrolling in a UIScrollView?

Short & Easy would be, just check the velocity value, if its greater than zero then its scrolling left else right:

func scrollViewWillEndDragging(scrollView: UIScrollView, withVelocity velocity: CGPoint, targetContentOffset: UnsafeMutablePointer<CGPoint>) {

    var targetOffset = Float(targetContentOffset.memory.x)
    println("TargetOffset: \(targetOffset)")
    println(velocity)

    if velocity.x < 0 {
        scrollDirection = -1 //scrolling left
    } else {
        scrollDirection = 1 //scrolling right
    }
}

PHP how to get the base domain/url?

You could use PHP's parse_url() function

function url($url) {
  $result = parse_url($url);
  return $result['scheme']."://".$result['host'];
}

Most efficient way to concatenate strings?

It's also important to point it out that you should use the + operator if you are concatenating string literals.

When you concatenate string literals or string constants by using the + operator, the compiler creates a single string. No run time concatenation occurs.

How to: Concatenate Multiple Strings (C# Programming Guide)

CSS: Fix row height

I haven't tried it but if you put a div in your table cell set so that it will have scrollbars if needed, then you could insert in there, with a fixed height on the div and it should keep your table row to a fixed height.

cannot convert 'std::basic_string<char>' to 'const char*' for argument '1' to 'int system(const char*)'

The system function requires const char *, and your expression is of the type std::string. You should write

string name = "john";
string system_str = " quickscan.exe resolution 300 selectscanner jpg showui showprogress filename '"+name+".jpg'";
system(system_str.c_str ());

How to use table variable in a dynamic sql statement?

Here is an example of using a dynamic T-SQL query and then extracting the results should you have more than one column of returned values (notice the dynamic table name):

DECLARE 
@strSQLMain nvarchar(1000),
@recAPD_number_key char(10),    
@Census_sub_code varchar(1),
@recAPD_field_name char(100),
@recAPD_table_name char(100),
@NUMBER_KEY varchar(10),

if object_id('[Permits].[dbo].[myTempAPD_Txt]') is not null 

    DROP TABLE [Permits].[dbo].[myTempAPD_Txt]

CREATE TABLE [Permits].[dbo].[myTempAPD_Txt]
(
    [MyCol1] char(10) NULL,
    [MyCol2] char(1) NULL,

)   
-- an example of what @strSQLMain is : @strSQLMain = SELECT @recAPD_number_key = [NUMBER_KEY], @Census_sub_code=TEXT_029 FROM APD_TXT0 WHERE Number_Key = '01-7212' 
SET @strSQLMain = ('INSERT INTO myTempAPD_Txt SELECT [NUMBER_KEY], '+ rtrim(@recAPD_field_name) +' FROM '+ rtrim(@recAPD_table_name) + ' WHERE Number_Key = '''+ rtrim(@Number_Key) +'''')      
EXEC (@strSQLMain)  
SELECT @recAPD_number_key = MyCol1, @Census_sub_code = MyCol2 from [Permits].[dbo].[myTempAPD_Txt]

DROP TABLE [Permits].[dbo].[myTempAPD_Txt]  

Pandas - Plotting a stacked Bar Chart

That should help

df.groupby(['NFF', 'ABUSE']).size().unstack().plot(kind='bar', stacked=True)

Can't concatenate 2 arrays in PHP

you may use operator . $array3 = $array1.$array2;

Import JSON file in React

try with export default DATA or module.exports = DATA

Sorting dropdown alphabetically in AngularJS

You should be able to use filter: orderBy

orderBy can accept a third option for the reverse flag.

<select ng-option="item.name for item in items | orderBy:'name':true"></select>

Here item is sorted by 'name' property in a reversed order. The 2nd argument can be any order function, so you can sort in any rule.

@see http://docs.angularjs.org/api/ng.filter:orderBy

Android: How do bluetooth UUIDs work?

The UUID stands for Universally Unique Identifier. UUID is an simple 128 bit digit which uniquely distributed across the world.

Bluetooth sends data over air and all nearby device can receive it. Let's suppose, sometimes you have to send some important files via Bluetooth and all near by devices can access it in range. So when you pair with the other devices, they simply share the UUID number and match before sharing the files. When you send any file then your device encrypt that file with appropriate device UUID and share over the network. Now all Bluetooth devices in the range can access the encrypt file but they required right UUID number. So Only right UUID devices have access to encrypt the file and others will reject cause of wrong UUID.

In short, you can use UUID as a secret password for sharing files between any two Bluetooth devices.

How Big can a Python List Get?

I got this from here on a x64 bit system: Python 3.7.0b5 (v3.7.0b5:abb8802389, May 31 2018, 01:54:01) [MSC v.1913 64 bit (AMD64)] on win32

enter image description here

Bootstrap col-md-offset-* not working

In bootstrap 3 the format is

col-md-6 col-md-offset-3

For the same grid in Bootstrap 4 the format is

col-md-6 offset-md-3

Calculate mean and standard deviation from a vector of samples in C++ using Boost

My answer is similar as Josh Greifer but generalised to sample covariance. Sample variance is just sample covariance but with the two inputs identical. This includes Bessel's correlation.

    template <class Iter> typename Iter::value_type cov(const Iter &x, const Iter &y)
    {
        double sum_x = std::accumulate(std::begin(x), std::end(x), 0.0);
        double sum_y = std::accumulate(std::begin(y), std::end(y), 0.0);

        double mx =  sum_x / x.size();
        double my =  sum_y / y.size();

        double accum = 0.0;

        for (auto i = 0; i < x.size(); i++)
        {
            accum += (x.at(i) - mx) * (y.at(i) - my);
        }

        return accum / (x.size() - 1);
    }

generate model using user:references vs user_id:integer

how does rails know that user_id is a foreign key referencing user?

Rails itself does not know that user_id is a foreign key referencing user. In the first command rails generate model Micropost user_id:integer it only adds a column user_id however rails does not know the use of the col. You need to manually put the line in the Micropost model

class Micropost < ActiveRecord::Base
  belongs_to :user
end

class User < ActiveRecord::Base
  has_many :microposts
end

the keywords belongs_to and has_many determine the relationship between these models and declare user_id as a foreign key to User model.

The later command rails generate model Micropost user:references adds the line belongs_to :user in the Micropost model and hereby declares as a foreign key.

FYI
Declaring the foreign keys using the former method only lets the Rails know about the relationship the models/tables have. The database is unknown about the relationship. Therefore when you generate the EER Diagrams using software like MySql Workbench you find that there is no relationship threads drawn between the models. Like in the following pic enter image description here

However, if you use the later method you find that you migration file looks like:

def change
    create_table :microposts do |t|
      t.references :user, index: true

      t.timestamps null: false
    end
    add_foreign_key :microposts, :users

Now the foreign key is set at the database level. and you can generate proper EER diagrams. enter image description here

Javascript split regex question

or just (anything but numbers):

date.split(/\D/);

How to fix System.NullReferenceException: Object reference not set to an instance of an object

If the problem is 100% here

EffectSelectorForm effectSelectorForm = new EffectSelectorForm(Effects);

There's only one possible explanation: property/variable "Effects" is not initialized properly... Debug your code to see what you pass to your objects.

EDIT after several hours

There were some problems:

  • MEF attribute [Import] didn't work as expected, so we replaced it for the time being with a manually populated List<>. While the collection was null, it was causing exceptions later in the code, when the method tried to get the type of the selected item and there was none.

  • several event handlers weren't wired up to control events

Some problems are still present, but I believe OP's original problem has been fixed. Other problems are not related to this one.

JavaScript REST client Library

You can use http://adodson.com/hello.js/ which has

  1. Rest API support
  2. Built in support for many sites google, facebook, dropbox
  3. It supports oAuth 1 and 2 support.

New self vs. new static

will I get the same results?

Not really. I don't know of a workaround for PHP 5.2, though.

What is the difference between new self and new static?

self refers to the same class in which the new keyword is actually written.

static, in PHP 5.3's late static bindings, refers to whatever class in the hierarchy you called the method on.

In the following example, B inherits both methods from A. The self invocation is bound to A because it's defined in A's implementation of the first method, whereas static is bound to the called class (also see get_called_class()).

class A {
    public static function get_self() {
        return new self();
    }

    public static function get_static() {
        return new static();
    }
}

class B extends A {}

echo get_class(B::get_self());  // A
echo get_class(B::get_static()); // B
echo get_class(A::get_self()); // A
echo get_class(A::get_static()); // A

Can I write or modify data on an RFID tag?

RFID tag has more standards. I have developed the RFID tag on Mifare card (ISO 14443A,B) and ISO 15693. Both of them, you can read/write or modify the data in the block data of RFID tag.

How to replace local branch with remote branch entirely in Git?

You can do as @Hugo of @Laurent said, or you can use git rebase to delete the commits you want to get rid off, if you know which ones. I tend to use git rebase -i head~N (where N is a number, allowing you to manipulate the last N commits) for this kind of operations.

Hive cast string to date dd-MM-yyyy

Let's say you have a column 'birth_day' in your table which is in string format, you should use the following query to filter using birth_day

date_Format(birth_day, 'yyyy-MM-dd')

You can use it in a query in the following way

select * from yourtable
where 
date_Format(birth_day, 'yyyy-MM-dd') = '2019-04-16';

Angular Material: mat-select not selecting default

I followed the above very carefully and still couldn't get the initial value selected.

The reason was that although my bound value was defined as a string in typescript, my backend API was returning a number.

Javascript loose typing simply changed the type at runtime (without error), which prevented selection the of the initial value.

Component

myBoundValue: string;

Template

<mat-select [(ngModel)]="myBoundValue">

Solution was to update the API to return a string value.

How to execute a file within the python interpreter?

From my view, the best way is:

import yourfile

and after modifying yourfile.py

reload(yourfile)   

or

import imp; 
imp.reload(yourfile) in python3

but this will make the function and classes looks like that: yourfile.function1, yourfile.class1.....

If you cannot accept those, the finally solution is:

reload(yourfile)
from yourfile import *

Android Studio - ADB Error - "...device unauthorized. Please check the confirmation dialog on your device."

1) Go to Phone Setting > Developer options > Revoke USB debugging.

2) Turn off USB debugging and Restart Again.

It will work definitely, in my case it worked.

How can I use Bash syntax in Makefile targets?

You can call bash directly, use the -c flag:

bash -c "diff <(sort file1) <(sort file2) > $@"

Of course, you may not be able to redirect to the variable $@, but when I tried to do this, I got -bash: $@: ambiguous redirect as an error message, so you may want to look into that before you get too into this (though I'm using bash 3.2.something, so maybe yours works differently).

Subtracting 2 lists in Python

A slightly different Vector class.

class Vector( object ):
    def __init__(self, *data):
        self.data = data
    def __repr__(self):
        return repr(self.data) 
    def __add__(self, other):
        return tuple( (a+b for a,b in zip(self.data, other.data) ) )  
    def __sub__(self, other):
        return tuple( (a-b for a,b in zip(self.data, other.data) ) )

Vector(1, 2, 3) - Vector(1, 1, 1)

C - freeing structs

Because you defined the struct as consisting of char arrays, the two strings are the structure and freeing the struct is sufficient, nor is there a way to free the struct but keep the arrays. For that case you would want to do something like struct { char *firstName, *lastName; }, but then you need to allocate memory for the names separately and handle the question of when to free that memory.

Aside: Is there a reason you want to keep the names after the struct has been freed?

MySQL: ignore errors when importing?

Use the --force (-f) flag on your mysql import. Rather than stopping on the offending statement, MySQL will continue and just log the errors to the console.

For example:

mysql -u userName -p -f -D dbName < script.sql

Create dataframe from a matrix

If you change your time column into row names, then you can use as.data.frame(as.table(mat)) for simple cases like this.

Example:

data <- c(0.1, 0.2, 0.3, 0.3, 0.4, 0.5)
dimnames <- list(time=c(0, 0.5, 1), name=c("C_0", "C_1"))
mat <- matrix(data, ncol=2, nrow=3, dimnames=dimnames)
as.data.frame(as.table(mat))
  time name Freq
1    0  C_0  0.1
2  0.5  C_0  0.2
3    1  C_0  0.3
4    0  C_1  0.3
5  0.5  C_1  0.4
6    1  C_1  0.5

In this case time and name are both factors. You may want to convert time back to numeric, or it may not matter.

Connection attempt failed with "ECONNREFUSED - Connection refused by server"

Use port number 22 (for sftp) instead of 21 (normal ftp). Solved this problem for me.

How to generate .angular-cli.json file in Angular Cli?

Since Angular version 6 .angular-cli.json is deprecated. That file was replaced by angular.json file which supports workspaces.

NPM global install "cannot find module"

I did this in simple way...

  1. Un-Install node from control panel [Windows 7]
  2. Install node again
  3. Install protractor npm install --global --verbose protractor
    Update web driver manager.

works fine for me.

Hope this helps you....

change figure size and figure format in matplotlib

The first part (setting the output size explictly) isn't too hard:

import matplotlib.pyplot as plt
list1 = [3,4,5,6,9,12]
list2 = [8,12,14,15,17,20]
fig = plt.figure(figsize=(4,3))
ax = fig.add_subplot(111)
ax.plot(list1, list2)
fig.savefig('fig1.png', dpi = 300)
fig.close()

But after a quick google search on matplotlib + tiff, I'm not convinced that matplotlib can make tiff plots. There is some mention of the GDK backend being able to do it.

One option would be to convert the output with a tool like imagemagick's convert.

(Another option is to wait around here until a real matplotlib expert shows up and proves me wrong ;-)

OnChange event handler for radio button (INPUT type="radio") doesn't work as one value

Easiest and power full way

read only radio inputs using getAttribute

_x000D_
_x000D_
document.addEventListener('input',(e)=>{

if(e.target.getAttribute('name')=="myRadios")
console.log(e.target.value)
})
_x000D_
<input type="radio" name="myRadios" value="1" /> 1
<input type="radio" name="myRadios" value="2" /> 2
_x000D_
_x000D_
_x000D_

print call stack in C or C++

Linux specific, TLDR:

  1. backtrace in glibc produces accurate stacktraces only when -lunwind is linked (undocumented platform-specific feature).
  2. To output function name, source file and line number use #include <elfutils/libdwfl.h> (this library is documented only in its header file). backtrace_symbols and backtrace_symbolsd_fd are least informative.

On modern Linux your can get the stacktrace addresses using function backtrace. The undocumented way to make backtrace produce more accurate addresses on popular platforms is to link with -lunwind (libunwind-dev on Ubuntu 18.04) (see the example output below). backtrace uses function _Unwind_Backtrace and by default the latter comes from libgcc_s.so.1 and that implementation is most portable. When -lunwind is linked it provides a more accurate version of _Unwind_Backtrace but this library is less portable (see supported architectures in libunwind/src).

Unfortunately, the companion backtrace_symbolsd and backtrace_symbols_fd functions have not been able to resolve the stacktrace addresses to function names with source file name and line number for probably a decade now (see the example output below).

However, there is another method to resolve addresses to symbols and it produces the most useful traces with function name, source file and line number. The method is to #include <elfutils/libdwfl.h>and link with -ldw (libdw-dev on Ubuntu 18.04).

Working C++ example (test.cc):

#include <stdexcept>
#include <iostream>
#include <cassert>
#include <cstdlib>
#include <string>

#include <boost/core/demangle.hpp>

#include <execinfo.h>
#include <elfutils/libdwfl.h>

struct DebugInfoSession {
    Dwfl_Callbacks callbacks = {};
    char* debuginfo_path = nullptr;
    Dwfl* dwfl = nullptr;

    DebugInfoSession() {
        callbacks.find_elf = dwfl_linux_proc_find_elf;
        callbacks.find_debuginfo = dwfl_standard_find_debuginfo;
        callbacks.debuginfo_path = &debuginfo_path;

        dwfl = dwfl_begin(&callbacks);
        assert(dwfl);

        int r;
        r = dwfl_linux_proc_report(dwfl, getpid());
        assert(!r);
        r = dwfl_report_end(dwfl, nullptr, nullptr);
        assert(!r);
        static_cast<void>(r);
    }

    ~DebugInfoSession() {
        dwfl_end(dwfl);
    }

    DebugInfoSession(DebugInfoSession const&) = delete;
    DebugInfoSession& operator=(DebugInfoSession const&) = delete;
};

struct DebugInfo {
    void* ip;
    std::string function;
    char const* file;
    int line;

    DebugInfo(DebugInfoSession const& dis, void* ip)
        : ip(ip)
        , file()
        , line(-1)
    {
        // Get function name.
        uintptr_t ip2 = reinterpret_cast<uintptr_t>(ip);
        Dwfl_Module* module = dwfl_addrmodule(dis.dwfl, ip2);
        char const* name = dwfl_module_addrname(module, ip2);
        function = name ? boost::core::demangle(name) : "<unknown>";

        // Get source filename and line number.
        if(Dwfl_Line* dwfl_line = dwfl_module_getsrc(module, ip2)) {
            Dwarf_Addr addr;
            file = dwfl_lineinfo(dwfl_line, &addr, &line, nullptr, nullptr, nullptr);
        }
    }
};

std::ostream& operator<<(std::ostream& s, DebugInfo const& di) {
    s << di.ip << ' ' << di.function;
    if(di.file)
        s << " at " << di.file << ':' << di.line;
    return s;
}

void terminate_with_stacktrace() {
    void* stack[512];
    int stack_size = ::backtrace(stack, sizeof stack / sizeof *stack);

    // Print the exception info, if any.
    if(auto ex = std::current_exception()) {
        try {
            std::rethrow_exception(ex);
        }
        catch(std::exception& e) {
            std::cerr << "Fatal exception " << boost::core::demangle(typeid(e).name()) << ": " << e.what() << ".\n";
        }
        catch(...) {
            std::cerr << "Fatal unknown exception.\n";
        }
    }

    DebugInfoSession dis;
    std::cerr << "Stacktrace of " << stack_size << " frames:\n";
    for(int i = 0; i < stack_size; ++i) {
        std::cerr << i << ": " << DebugInfo(dis, stack[i]) << '\n';
    }
    std::cerr.flush();

    std::_Exit(EXIT_FAILURE);
}

int main() {
    std::set_terminate(terminate_with_stacktrace);
    throw std::runtime_error("test exception");
}

Compiled on Ubuntu 18.04.4 LTS with gcc-8.3:

g++ -o test.o -c -m{arch,tune}=native -std=gnu++17 -W{all,extra,error} -g -Og -fstack-protector-all test.cc
g++ -o test -g test.o -ldw -lunwind

Outputs:

Fatal exception std::runtime_error: test exception.
Stacktrace of 7 frames:
0: 0x55f3837c1a8c terminate_with_stacktrace() at /home/max/src/test/test.cc:76
1: 0x7fbc1c845ae5 <unknown>
2: 0x7fbc1c845b20 std::terminate()
3: 0x7fbc1c845d53 __cxa_throw
4: 0x55f3837c1a43 main at /home/max/src/test/test.cc:103
5: 0x7fbc1c3e3b96 __libc_start_main at ../csu/libc-start.c:310
6: 0x55f3837c17e9 _start

When no -lunwind is linked, it produces a less accurate stacktrace:

0: 0x5591dd9d1a4d terminate_with_stacktrace() at /home/max/src/test/test.cc:76
1: 0x7f3c18ad6ae6 <unknown>
2: 0x7f3c18ad6b21 <unknown>
3: 0x7f3c18ad6d54 <unknown>
4: 0x5591dd9d1a04 main at /home/max/src/test/test.cc:103
5: 0x7f3c1845cb97 __libc_start_main at ../csu/libc-start.c:344
6: 0x5591dd9d17aa _start

For comparison, backtrace_symbols_fd output for the same stacktrace is least informative:

/home/max/src/test/debug/gcc/test(+0x192f)[0x5601c5a2092f]
/usr/lib/x86_64-linux-gnu/libstdc++.so.6(+0x92ae5)[0x7f95184f5ae5]
/usr/lib/x86_64-linux-gnu/libstdc++.so.6(_ZSt9terminatev+0x10)[0x7f95184f5b20]
/usr/lib/x86_64-linux-gnu/libstdc++.so.6(__cxa_throw+0x43)[0x7f95184f5d53]
/home/max/src/test/debug/gcc/test(+0x1ae7)[0x5601c5a20ae7]
/lib/x86_64-linux-gnu/libc.so.6(__libc_start_main+0xe6)[0x7f9518093b96]
/home/max/src/test/debug/gcc/test(+0x1849)[0x5601c5a20849]

In a production version (as well as C language version) you may like to make this code extra robust by replacing boost::core::demangle, std::string and std::cout with their underlying calls.

You can also override __cxa_throw to capture the stacktrace when an exception is thrown and print it when the exception is caught. By the time it enters catch block the stack has been unwound, so it is too late to call backtrace, and this is why the stack must be captured on throw which is implemented by function __cxa_throw. Note that in a multi-threaded program __cxa_throw can be called simultaneously by multiple threads, so that if it captures the stacktrace into a global array that must be thread_local.

Modifying a query string without reloading the page

If you are looking for Hash modification, your solution works ok. However, if you want to change the query, you can use the pushState, as you said. Here it is an example that might help you to implement it properly. I tested and it worked fine:

if (history.pushState) {
    var newurl = window.location.protocol + "//" + window.location.host + window.location.pathname + '?myNewUrlQuery=1';
    window.history.pushState({path:newurl},'',newurl);
}

It does not reload the page, but it only allows you to change the URL query. You would not be able to change the protocol or the host values. And of course that it requires modern browsers that can process HTML5 History API.

For more information:

http://diveintohtml5.info/history.html

https://developer.mozilla.org/en-US/docs/Web/Guide/API/DOM/Manipulating_the_browser_history

How to make borders collapse (on a div)?

You could also use negative margins:

_x000D_
_x000D_
.column {_x000D_
  float: left;_x000D_
  overflow: hidden;_x000D_
  width: 120px;_x000D_
}_x000D_
.cell {_x000D_
  border: 1px solid red;_x000D_
  width: 120px;_x000D_
  height: 20px;_x000D_
  box-sizing: border-box;_x000D_
}_x000D_
.cell:not(:first-child) {_x000D_
  margin-top: -1px;_x000D_
}_x000D_
.column:not(:first-child) > .cell {_x000D_
  margin-left: -1px;_x000D_
}
_x000D_
<div class="container">_x000D_
  <div class="column">_x000D_
    <div class="cell"></div>_x000D_
    <div class="cell"></div>_x000D_
    <div class="cell"></div>_x000D_
  </div>_x000D_
  <div class="column">_x000D_
    <div class="cell"></div>_x000D_
    <div class="cell"></div>_x000D_
    <div class="cell"></div>_x000D_
  </div>_x000D_
  <div class="column">_x000D_
    <div class="cell"></div>_x000D_
    <div class="cell"></div>_x000D_
    <div class="cell"></div>_x000D_
  </div>_x000D_
  <div class="column">_x000D_
    <div class="cell"></div>_x000D_
    <div class="cell"></div>_x000D_
    <div class="cell"></div>_x000D_
  </div>_x000D_
  <div class="column">_x000D_
    <div class="cell"></div>_x000D_
    <div class="cell"></div>_x000D_
    <div class="cell"></div>_x000D_
  </div>_x000D_
</div>
_x000D_
_x000D_
_x000D_

Change Schema Name Of Table In SQL

Through SSMS, I created a new schema by:

  • Clicking the Security folder in the Object Explorer within my server,
  • right clicked Schemas
  • Selected "New Schema..."
  • Named my new schema (exe in your case)
  • Hit OK

I found this post to change the schema, but was also getting the same permissions error when trying to change to the new schema. I have several databases listed in my SSMS, so I just tried specifying the database and it worked:

USE (yourservername)  
ALTER SCHEMA exe TRANSFER dbo.Employees 

Delete all nodes and relationships in neo4j 1.8

It will do the trick..

Match (n)-[r]-()
Delete n,r;

Jenkins - Configure Jenkins to poll changes in SCM

I believe best practice these days is H/5 * * * *, which means every 5 minutes with a hashing factor to avoid all jobs starting at EXACTLY the same time.

Scope 'session' is not active for the current thread; IllegalStateException: No thread-bound request found

If anyone else stuck on same point, following solved my problem.

In web.xml

 <listener>
            <listener-class>
                    org.springframework.web.context.request.RequestContextListener 
            </listener-class>
  </listener>

In Session component

@Component
@Scope(value = "session",  proxyMode = ScopedProxyMode.TARGET_CLASS)

In pom.xml

    <dependency>
        <groupId>cglib</groupId>
        <artifactId>cglib</artifactId>
        <version>3.1</version>
    </dependency>

for or while loop to do something n times

This is lighter weight than xrange (and the while loop) since it doesn't even need to create the int objects. It also works equally well in Python2 and Python3

from itertools import repeat
for i in repeat(None, 10):
    do_sth()

403 Forbidden vs 401 Unauthorized HTTP responses

This is an older question, but one option that was never really brought up was to return a 404. From a security perspective, the highest voted answer suffers from a potential information leakage vulnerability. Say, for instance, that the secure web page in question is a system admin page, or perhaps more commonly, is a record in a system that the user doesn't have access to. Ideally you wouldn't want a malicious user to even know that there's a page / record there, let alone that they don't have access. When I'm building something like this, I'll try to record unauthenticate / unauthorized requests in an internal log, but return a 404.

OWASP has some more information about how an attacker could use this type of information as part of an attack.

Proper use of errors

Simple solution to emit and show message by Exception.

try {
  throw new TypeError("Error message");
}
catch (e){
  console.log((<Error>e).message);//conversion to Error type
}

Caution

Above is not a solution if we don't know what kind of error can be emitted from the block. In such cases type guards should be used and proper handling for proper error should be done - take a look on @Moriarty answer.

Count all values in a matrix greater than a value

To count the number of values larger than x in any numpy array you can use:

n = len(matrix[matrix > x])

The boolean indexing returns an array that contains only the elements where the condition (matrix > x) is met. Then len() counts these values.

Find intersection of two nested lists?

You should flatten using this code ( taken from http://kogs-www.informatik.uni-hamburg.de/~meine/python_tricks ), the code is untested, but I'm pretty sure it works:


def flatten(x):
    """flatten(sequence) -> list

    Returns a single, flat list which contains all elements retrieved
    from the sequence and all recursively contained sub-sequences
    (iterables).

    Examples:
    >>> [1, 2, [3,4], (5,6)]
    [1, 2, [3, 4], (5, 6)]
    >>> flatten([[[1,2,3], (42,None)], [4,5], [6], 7, MyVector(8,9,10)])
    [1, 2, 3, 42, None, 4, 5, 6, 7, 8, 9, 10]"""

    result = []
    for el in x:
        #if isinstance(el, (list, tuple)):
        if hasattr(el, "__iter__") and not isinstance(el, basestring):
            result.extend(flatten(el))
        else:
            result.append(el)
    return result

After you had flattened the list, you perform the intersection in the usual way:


c1 = [1, 6, 7, 10, 13, 28, 32, 41, 58, 63]
c2 = [[13, 17, 18, 21, 32], [7, 11, 13, 14, 28], [1, 5, 6, 8, 15, 16]]

def intersect(a, b):
     return list(set(a) & set(b))

print intersect(flatten(c1), flatten(c2))

How to change permissions for a folder and its subfolders/files in one step?

chmod -R 755 directory_name works, but how would you keep new files to 755 also? The file's permissions becomes the default permission.

How to have image and text side by side

Your h4 has some crazy margin on it, so remove it

h4 {
    margin:0px;
}

http://jsfiddle.net/qMdfC/2/

edit:

http://jsfiddle.net/qMdfC/6/

for the 0 minutes text, added a float left to the first div, but personally i'd just combine them, although you may have reasons not to.

get next and previous day with PHP

it is enough to call it this way:

<a href="home.php?date=<?= date('Y-m-d', strtotime('-1 day')) ?>" class="prev_day" title="Previous Day" ></a>
<a href="home.php?date=<?= date('Y-m-d', strtotime('+1 day')) ?>" class="next_day" title="Next Day" ></a>

Also see the documentation.

How do you add an in-app purchase to an iOS application?

I know I am quite late to post this, but I share similar experience when I learned the ropes of IAP model.

In-app purchase is one of the most comprehensive workflow in iOS implemented by Storekit framework. The entire documentation is quite clear if you patience to read it, but is somewhat advanced in nature of technicality.

To summarize:

1 - Request the products - use SKProductRequest & SKProductRequestDelegate classes to issue request for Product IDs and receive them back from your own itunesconnect store.

These SKProducts should be used to populate your store UI which the user can use to buy a specific product.

2 - Issue payment request - use SKPayment & SKPaymentQueue to add payment to the transaction queue.

3 - Monitor transaction queue for status update - use SKPaymentTransactionObserver Protocol's updatedTransactions method to monitor status:

SKPaymentTransactionStatePurchasing - don't do anything
SKPaymentTransactionStatePurchased - unlock product, finish the transaction
SKPaymentTransactionStateFailed - show error, finish the transaction
SKPaymentTransactionStateRestored - unlock product, finish the transaction

4 - Restore button flow - use SKPaymentQueue's restoreCompletedTransactions to accomplish this - step 3 will take care of the rest, along with SKPaymentTransactionObserver's following methods:

paymentQueueRestoreCompletedTransactionsFinished
restoreCompletedTransactionsFailedWithError

Here is a step by step tutorial (authored by me as a result of my own attempts to understand it) that explains it. At the end it also provides code sample that you can directly use.

Here is another one I created to explain certain things that only text could describe in better manner.

SVN: Folder already under version control but not comitting?

I found a solution in case you have installed Eclipse(Luna) with the SVN Client JavaHL(JNI) 1.8.13 and Tortoise:

Open Eclipse: First try to add the project / maven module to Version Control (Project -> Context Menu -> Team -> Add to Version Control)

You will see the following Eclipse error message:

org.apache.subversion.javahl.ClientException: Entry already exists svn: 'PathToYouProject' is already under version control

After that you have to open your workspace directory in your explorer, select your project and resolve it via Tortoise (Project -> Context Menu -> TortoiseSVN -> Resolve)

You will see the following message dialog: "File list is empty"

Press cancel and refresh the project in Eclipse. Your project should be under version control again.

Unfortunately it is not possible to resolve more the one project at the same time ... you don't have to delete anything but depending on the size of your project it could be a little bit laborious.

How can I read a text file in Android?

Try this :

I assume your text file is on sd card

    //Find the directory for the SD Card using the API
//*Don't* hardcode "/sdcard"
File sdcard = Environment.getExternalStorageDirectory();

//Get the text file
File file = new File(sdcard,"file.txt");

//Read text from file
StringBuilder text = new StringBuilder();

try {
    BufferedReader br = new BufferedReader(new FileReader(file));
    String line;

    while ((line = br.readLine()) != null) {
        text.append(line);
        text.append('\n');
    }
    br.close();
}
catch (IOException e) {
    //You'll need to add proper error handling here
}

//Find the view by its id
TextView tv = (TextView)findViewById(R.id.text_view);

//Set the text
tv.setText(text.toString());

following links can also help you :

How can I read a text file from the SD card in Android?

How to read text file in Android?

Android read text raw resource file

INSERT INTO @TABLE EXEC @query with SQL Server 2000

N.B. - this question and answer relate to the 2000 version of SQL Server. In later versions, the restriction on INSERT INTO @table_variable ... EXEC ... were lifted and so it doesn't apply for those later versions.


You'll have to switch to a temp table:

CREATE TABLE #tmp (code varchar(50), mount money)
DECLARE @q nvarchar(4000)
SET @q = 'SELECT coa_code, amount FROM T_Ledger_detail'

INSERT INTO  #tmp (code, mount)
EXEC sp_executesql (@q)

SELECT * from #tmp

From the documentation:

A table variable behaves like a local variable. It has a well-defined scope, which is the function, stored procedure, or batch in which it is declared.

Within its scope, a table variable may be used like a regular table. It may be applied anywhere a table or table expression is used in SELECT, INSERT, UPDATE, and DELETE statements. However, table may not be used in the following statements:

INSERT INTO table_variable EXEC stored_procedure

SELECT select_list INTO table_variable statements.

How do I compare strings in Java?

== tests for reference equality (whether they are the same object).

.equals() tests for value equality (whether they are logically "equal").

Objects.equals() checks for null before calling .equals() so you don't have to (available as of JDK7, also available in Guava).

Consequently, if you want to test whether two strings have the same value you will probably want to use Objects.equals().

// These two have the same value
new String("test").equals("test") // --> true 

// ... but they are not the same object
new String("test") == "test" // --> false 

// ... neither are these
new String("test") == new String("test") // --> false 

// ... but these are because literals are interned by 
// the compiler and thus refer to the same object
"test" == "test" // --> true 

// ... string literals are concatenated by the compiler
// and the results are interned.
"test" == "te" + "st" // --> true

// ... but you should really just call Objects.equals()
Objects.equals("test", new String("test")) // --> true
Objects.equals(null, "test") // --> false
Objects.equals(null, null) // --> true

You almost always want to use Objects.equals(). In the rare situation where you know you're dealing with interned strings, you can use ==.

From JLS 3.10.5. String Literals:

Moreover, a string literal always refers to the same instance of class String. This is because string literals - or, more generally, strings that are the values of constant expressions (§15.28) - are "interned" so as to share unique instances, using the method String.intern.

Similar examples can also be found in JLS 3.10.5-1.

Other Methods To Consider

String.equalsIgnoreCase() value equality that ignores case. Beware, however, that this method can have unexpected results in various locale-related cases, see this question.

String.contentEquals() compares the content of the String with the content of any CharSequence (available since Java 1.5). Saves you from having to turn your StringBuffer, etc into a String before doing the equality comparison, but leaves the null checking to you.

It says that TypeError: document.getElementById(...) is null

I got the same error. In my case I had multiple div with same id in a page. I renamed the another id of the div used and fixed the issue.

So confirm whether the element:

  • exists with id
  • doesn't have duplicate with id
  • confirm whether the script is called

Difference between ${} and $() in Bash

  1. your understanding is right. For detailed info on {} see bash ref - parameter expansion

  2. 'for' and 'while' have different syntax and offer different styles of programmer control for an iteration. Most non-asm languages offer a similar syntax.

With while, you would probably write i=0; while [ $i -lt 10 ]; do echo $i; i=$(( i + 1 )); done in essence manage everything about the iteration yourself

Apache POI Excel - how to configure columns to be expanded?

After you have added all your data to the sheet, you can call autoSizeColumn(int column) on your sheet to autofit the columns to the proper size

Here is a link to the API.

See this post for more reference Problem in fitting the excel cell size to the size of the content when using apache poi

How generate unique Integers based on GUIDs

The GetHashCode function is specifically designed to create a well distributed range of integers with a low probability of collision, so for this use case is likely to be the best you can do.

But, as I'm sure you're aware, hashing 128 bits of information into 32 bits of information throws away a lot of data, so there will almost certainly be collisions if you have a sufficiently large number of GUIDs.

Interpreting "condition has length > 1" warning from `if` function

Just adding a point to the whole discussion as to why this warning comes up (It wasn't clear to me before). The reason one gets this is as mentioned before is because 'a' in this case is a vector and the inequality 'a>0' produces another vector of TRUE and FALSE (where 'a' is >0 or not).

If you would like to instead test if any value of 'a>0', you can use functions - 'any' or 'all'

Best

Array initializing in Scala

To initialize an array filled with zeros, you can use:

> Array.fill[Byte](5)(0)
Array(0, 0, 0, 0, 0)

This is equivalent to Java's new byte[5].

How can I pair socks from a pile efficiently?

Two lines of thinking, the speed it takes to find any match, versus the speed it takes to find all matches compared to the storage.

For the second case, I wanted to point out a GPU paralleled version which queries the socks for all matches.

If you have multiple properties for which to match, you can make use of grouped tuples and fancier zip iterators and the transform functions of thrust, for simplicity sake though here is a simple GPU based query:

//test.cu
#include <thrust/device_vector.h>
#include <thrust/sequence.h>
#include <thrust/copy.h>
#include <thrust/count.h>
#include <thrust/remove.h>
#include <thrust/random.h>
#include <iostream>
#include <iterator>
#include <string>

// Define some types for pseudo code readability
typedef thrust::device_vector<int> GpuList;
typedef GpuList::iterator          GpuListIterator;

template <typename T>
struct ColoredSockQuery : public thrust::unary_function<T,bool>
{
    ColoredSockQuery( int colorToSearch )
    { SockColor = colorToSearch; }

    int SockColor;

    __host__ __device__
    bool operator()(T x)
    {
        return x == SockColor;
    }
};


struct GenerateRandomSockColor
{
    float lowBounds, highBounds;

    __host__ __device__
    GenerateRandomSockColor(int _a= 0, int _b= 1) : lowBounds(_a), highBounds(_b) {};

    __host__ __device__
    int operator()(const unsigned int n) const
    {
        thrust::default_random_engine rng;
        thrust::uniform_real_distribution<float> dist(lowBounds, highBounds);
        rng.discard(n);
        return dist(rng);
    }
};

template <typename GpuListIterator>
void PrintSocks(const std::string& name, GpuListIterator first, GpuListIterator last)
{
    typedef typename std::iterator_traits<GpuListIterator>::value_type T;

    std::cout << name << ": ";
    thrust::copy(first, last, std::ostream_iterator<T>(std::cout, " "));
    std::cout << "\n";
}

int main()
{
    int numberOfSocks = 10000000;
    GpuList socks(numberOfSocks);
    thrust::transform(thrust::make_counting_iterator(0),
                      thrust::make_counting_iterator(numberOfSocks),
                      socks.begin(),
                      GenerateRandomSockColor(0, 200));

    clock_t start = clock();

    GpuList sortedSocks(socks.size());
    GpuListIterator lastSortedSock = thrust::copy_if(socks.begin(),
                                                     socks.end(),
                                                     sortedSocks.begin(),
                                                     ColoredSockQuery<int>(2));
    clock_t stop = clock();

    PrintSocks("Sorted Socks: ", sortedSocks.begin(), lastSortedSock);

    double elapsed = (double)(stop - start) * 1000.0 / CLOCKS_PER_SEC;
    std::cout << "Time elapsed in ms: " << elapsed << "\n";

    return 0;
}

    //nvcc -std=c++11 -o test test.cu

Run time for 10 million socks: 9 ms

Storing C++ template function definitions in a .CPP file

That is a standard way to define template functions. I think there are three methods I read for defining templates. Or probably 4. Each with pros and cons.

  1. Define in class definition. I don't like this at all because I think class definitions are strictly for reference and should be easy to read. However it is much less tricky to define templates in class than outside. And not all template declarations are on the same level of complexity. This method also makes the template a true template.

  2. Define the template in the same header, but outside of the class. This is my preferred way most of the times. It keeps your class definition tidy, the template remains a true template. It however requires full template naming which can be tricky. Also, your code is available to all. But if you need your code to be inline this is the only way. You can also accomplish this by creating a .INL file at the end of your class definitions.

  3. Include the header.h and implementation.CPP into your main.CPP. I think that's how its done. You won't have to prepare any pre instantiations, it will behave like a true template. The problem I have with it is that it is not natural. We don't normally include and expect to include source files. I guess since you included the source file, the template functions can be inlined.

  4. This last method, which was the posted way, is defining the templates in a source file, just like number 3; but instead of including the source file, we pre instantiate the templates to ones we will need. I have no problem with this method and it comes in handy sometimes. We have one big code, it cannot benefit from being inlined so just put it in a CPP file. And if we know common instantiations and we can predefine them. This saves us from writing basically the same thing 5, 10 times. This method has the benefit of keeping our code proprietary. But I don't recommend putting tiny, regularly used functions in CPP files. As this will reduce the performance of your library.

Note, I am not aware of the consequences of a bloated obj file.

Best practices for API versioning?

There are a few places you can do versioning in a REST API:

  1. As noted, in the URI. This can be tractable and even esthetically pleasing if redirects and the like are used well.

  2. In the Accepts: header, so the version is in the filetype. Like 'mp3' vs 'mp4'. This will also work, though IMO it works a bit less nicely than...

  3. In the resource itself. Many file formats have their version numbers embedded in them, typically in the header; this allows newer software to 'just work' by understanding all existing versions of the filetype while older software can punt if an unsupported (newer) version is specified. In the context of a REST API, it means that your URIs never have to change, just your response to the particular version of data you were handed.

I can see reasons to use all three approaches:

  1. if you like doing 'clean sweep' new APIs, or for major version changes where you want such an approach.
  2. if you want the client to know before it does a PUT/POST whether it's going to work or not.
  3. if it's okay if the client has to do its PUT/POST to find out if it's going to work.

Persistent invalid graphics state error when using ggplot2

try to get out grafics with x11() or win.graph() and solve this trouble.

Add element to a JSON file?

One possible issue I see is you set your JSON unconventionally within an array/list object. I would recommend using JSON in its most accepted form, i.e.:

test_json = { "a": 1, "b": 2}

Once you do this, adding a json element only involves the following line:

test_json["c"] = 3

This will result in:

{'a': 1, 'b': 2, 'c': 3}

Afterwards, you can add that json back into an array or a list of that is desired.

Does not contain a static 'main' method suitable for an entry point

I too have faced this problem. Then I realized that I was choosing Console Application(Package) rather than Console Application.

Transaction isolation levels relation with locks on table

The locks are always taken at DB level:-

Oracle official Document:- To avoid conflicts during a transaction, a DBMS uses locks, mechanisms for blocking access by others to the data that is being accessed by the transaction. (Note that in auto-commit mode, where each statement is a transaction, locks are held for only one statement.) After a lock is set, it remains in force until the transaction is committed or rolled back. For example, a DBMS could lock a row of a table until updates to it have been committed. The effect of this lock would be to prevent a user from getting a dirty read, that is, reading a value before it is made permanent. (Accessing an updated value that has not been committed is considered a dirty read because it is possible for that value to be rolled back to its previous value. If you read a value that is later rolled back, you will have read an invalid value.)

How locks are set is determined by what is called a transaction isolation level, which can range from not supporting transactions at all to supporting transactions that enforce very strict access rules.

One example of a transaction isolation level is TRANSACTION_READ_COMMITTED, which will not allow a value to be accessed until after it has been committed. In other words, if the transaction isolation level is set to TRANSACTION_READ_COMMITTED, the DBMS does not allow dirty reads to occur. The interface Connection includes five values that represent the transaction isolation levels you can use in JDBC.

How to use executeReader() method to retrieve the value of just one cell

It is not recommended to use DataReader and Command.ExecuteReader to get just one value from the database. Instead, you should use Command.ExecuteScalar as following:

String sql = "SELECT ColumnNumber FROM learer WHERE learer.id = " + index;
SqlCommand cmd = new SqlCommand(sql,conn);
learerLabel.Text = (String) cmd.ExecuteScalar();

Here is more information about Connecting to database and managing data.

Maximum Java heap size of a 32-bit JVM on a 64-bit OS

As to why a 32-bit JVM is used instead of a 64-bit one, the reason is not technical but rather administrative/bureaucratic ...

When I was working for BEA, we found that the average application actually ran slower in a 64-bit JVM, then it did when running in a 32-bit JVM. In some cases, the performance hit was as high as 25% slower. So, unless your application really needs all that extra memory, you were better off setting up more 32-bit servers.

As I recall, the three most common technical justifications for using a 64-bit that BEA professional services personnel ran into were:

  1. The application was manipulating multiple massive images,
  2. The application was doing massive number crunching,
  3. The application had a memory leak, the customer was the prime on a government contract, and they didn't want to take the time and the expense of tracking down the memory leak. (Using a massive memory heap would increase the MTBF and the prime would still get paid)

.

No value accessor for form control with name: 'recipient'

You should add the ngDefaultControl attribute to your input like this:

<md-input
    [(ngModel)]="recipient"
    name="recipient"
    placeholder="Name"
    class="col-sm-4"
    (blur)="addRecipient(recipient)"
    ngDefaultControl>
</md-input>

Taken from comments in this post:

angular2 rc.5 custom input, No value accessor for form control with unspecified name

Note: For later versions of @angular/material:

Nowadays you should instead write:

<md-input-container>
    <input
        mdInput
        [(ngModel)]="recipient"
        name="recipient"
        placeholder="Name"
        (blur)="addRecipient(recipient)">
</md-input-container>

See https://material.angular.io/components/input/overview

Why doesn't adding CORS headers to an OPTIONS route allow browsers to access my API?

Below worked for me, hope it helps someone!

const express = require('express');
const cors = require('cors');
let app = express();

app.use(cors({ origin: true }));

Got reference from https://expressjs.com/en/resources/middleware/cors.html#configuring-cors

How do I combine a background-image and CSS3 gradient on the same element?

If you also want to set background position for your image, than you can use this:

background-color: #444; // fallback
background: url('PATH-TO-IMG') center center no-repeat; // fallback

background: url('PATH-TO-IMG') center center no-repeat, -moz-linear-gradient(top, @startColor, @endColor); // FF 3.6+
background: url('PATH-TO-IMG') center center no-repeat, -webkit-gradient(linear, 0 0, 0 100%, from(@startColor), to(@endColor)); // Safari 4+, Chrome 2+
background: url('PATH-TO-IMG') center center no-repeat, -webkit-linear-gradient(top, @startColor, @endColor); // Safari 5.1+, Chrome 10+
background: url('PATH-TO-IMG') center center no-repeat, -o-linear-gradient(top, @startColor, @endColor); // Opera 11.10
background: url('PATH-TO-IMG') center center no-repeat, linear-gradient(to bottom, @startColor, @endColor); // Standard, IE10

or you can also create a LESS mixin (bootstrap style):

#gradient {
    .vertical-with-image(@startColor: #555, @endColor: #333, @image) {
        background-color: mix(@startColor, @endColor, 60%); // fallback
        background-image: @image; // fallback

        background: @image, -moz-linear-gradient(top, @startColor, @endColor); // FF 3.6+
        background: @image, -webkit-gradient(linear, 0 0, 0 100%, from(@startColor), to(@endColor)); // Safari 4+, Chrome 2+
        background: @image, -webkit-linear-gradient(top, @startColor, @endColor); // Safari 5.1+, Chrome 10+
        background: @image, -o-linear-gradient(top, @startColor, @endColor); // Opera 11.10
        background: @image, linear-gradient(to bottom, @startColor, @endColor); // Standard, IE10
    }
}

Rounding a number to the nearest 5 or 10 or X

For a strict Visual Basic approach, you can convert the floating-point value to an integer to round to said integer. VB is one of the rare languages that rounds on type conversion (most others simply truncate.)

Multiples of 5 or x can be done simply by dividing before and multiplying after the round.

If you want to round and keep decimal places, Math.round(n, d) would work.

Getting byte array through input type = file

_x000D_
_x000D_
$(document).ready(function(){_x000D_
    (function (document) {_x000D_
  var input = document.getElementById("files"),_x000D_
  output = document.getElementById("result"),_x000D_
  fileData; // We need fileData to be visible to getBuffer._x000D_
_x000D_
  // Eventhandler for file input. _x000D_
  function openfile(evt) {_x000D_
    var files = input.files;_x000D_
    // Pass the file to the blob, not the input[0]._x000D_
    fileData = new Blob([files[0]]);_x000D_
    // Pass getBuffer to promise._x000D_
    var promise = new Promise(getBuffer);_x000D_
    // Wait for promise to be resolved, or log error._x000D_
    promise.then(function(data) {_x000D_
      // Here you can pass the bytes to another function._x000D_
      output.innerHTML = data.toString();_x000D_
      console.log(data);_x000D_
    }).catch(function(err) {_x000D_
      console.log('Error: ',err);_x000D_
    });_x000D_
  }_x000D_
_x000D_
  /* _x000D_
    Create a function which will be passed to the promise_x000D_
    and resolve it when FileReader has finished loading the file._x000D_
  */_x000D_
  function getBuffer(resolve) {_x000D_
    var reader = new FileReader();_x000D_
    reader.readAsArrayBuffer(fileData);_x000D_
    reader.onload = function() {_x000D_
      var arrayBuffer = reader.result_x000D_
      var bytes = new Uint8Array(arrayBuffer);_x000D_
      resolve(bytes);_x000D_
    }_x000D_
  }_x000D_
_x000D_
  // Eventlistener for file input._x000D_
  input.addEventListener('change', openfile, false);_x000D_
}(document));_x000D_
});
_x000D_
<!DOCTYPE html>_x000D_
<html>_x000D_
<head>_x000D_
<script src="https://ajax.googleapis.com/ajax/libs/jquery/3.3.1/jquery.min.js"></script>_x000D_
</head>_x000D_
<body>_x000D_
_x000D_
<input type="file" id="files"/>_x000D_
<div id="result"></div>_x000D_
</body>_x000D_
</html>
_x000D_
_x000D_
_x000D_

Why does "pip install" inside Python raise a SyntaxError?

Initially I too faced this same problem, I installed python and when I run pip command it used to throw me an error like shown in pic below.

enter image description here

Make Sure pip path is added in environmental variables. For me, the python and pip installation path is::
Python: C:\Users\fhhz\AppData\Local\Programs\Python\Python38\
pip: C:\Users\fhhz\AppData\Local\Programs\Python\Python38\Scripts
Both these paths were added to path in environmental variables.

Now Open a new cmd window and type pip, you should be seeing a screen as below.

enter image description here

Now type pip install <<package-name>>. Here I'm installing package spyder so my command line statement will be as pip install spyder and here goes my running screen..

enter image description here

and I hope we are done with this!!

What is the difference between a pandas Series and a single-column DataFrame?

Quoting the Pandas docs

pandas.DataFrame(data=None, index=None, columns=None, dtype=None, copy=False)

Two-dimensional size-mutable, potentially heterogeneous tabular data structure with labeled axes (rows and columns). Arithmetic operations align on both row and column labels. Can be thought of as a dict-like container for Series objects. The primary pandas data structure.

So, the Series is the data structure for a single column of a DataFrame, not only conceptually, but literally, i.e. the data in a DataFrame is actually stored in memory as a collection of Series.

Analogously: We need both lists and matrices, because matrices are built with lists. Single row matricies, while equivalent to lists in functionality still cannot exist without the list(s) they're composed of.

They both have extremely similar APIs, but you'll find that DataFrame methods always cater to the possibility that you have more than one column. And, of course, you can always add another Series (or equivalent object) to a DataFrame, while adding a Series to another Series involves creating a DataFrame.

MySQL default datetime through phpmyadmin

I don't think you can achieve that with mysql date. You have to use timestamp or try this approach..

CREATE TRIGGER table_OnInsert BEFORE INSERT ON `DB`.`table`
FOR EACH ROW SET NEW.dateColumn = IFNULL(NEW.dateColumn, NOW());

What datatype to use when storing latitude and longitude data in SQL databases?

You should take a look at the new Spatial data-types that were introduced in SQL Server 2008. They are specifically designed this kind of task and make indexing and querying the data much easier and more efficient.

http://msdn.microsoft.com/en-us/library/bb933876(v=sql.105).aspx

Class 'ViewController' has no initializers in swift

Replace var appDelegate : AppDelegate? with let appDelegate = UIApplication.sharedApplication().delegate as hinted on the second commented line in viewDidLoad().

The keyword "optional" refers exactly to the use of ?, see this for more details.

cannot resolve symbol javafx.application in IntelliJ Idea IDE

You might have a lower project language level than your JDK.

Check if: "Projeckt structure/project/Project-> language level" is lower than your JDK. I had the same problem with JDK 9 and the language level was per default set to 6.

I set the Project Language Level to 9 and everything worked fine after that.

You might have the same issue.

Apply global variable to Vuejs

If the global variable should not be written to by anything, including Vuejs, you can use Object.freeze to freeze your object. Adding it to Vue's viewmodel won't unfreeze it. Another option is to provide Vuejs with a frozen copy of the object, if the object is intended to be written globally but just not by Vue: var frozenCopy = Object.freeze(Object.assign({}, globalObject))

Pandas Split Dataframe into two Dataframes at a specific row

I generally use array split because it's easier simple syntax and scales better with more than 2 partitions.

import numpy as np
partitions = 2
dfs = np.array_split(df, partitions)

np.split(df, [100,200,300], axis=0] wants explicit index numbers which may or may not be desirable.

How to download a file from a website in C#

Also you can use DownloadFileAsync method in WebClient class. It downloads to a local file the resource with the specified URI. Also this method does not block the calling thread.

Sample:

    webClient.DownloadFileAsync(new Uri("http://www.example.com/file/test.jpg"), "test.jpg");

For more information:

http://csharpexamples.com/download-files-synchronous-asynchronous-url-c/

MySQL/SQL: Group by date only on a Datetime column

Or:

SELECT SUM(foo), DATE(mydate) mydate FROM a_table GROUP BY mydate;

More efficient (I think.) Because you don't have to cast mydate twice per row.

How to add a new line of text to an existing file in Java?

On line 2 change new FileWriter(my_file_name) to new FileWriter(my_file_name, true) so you're appending to the file rather than overwriting.

File f = new File("/path/of/the/file");
        try {
            BufferedWriter bw = new BufferedWriter(new FileWriter(f, true));
            bw.append(line);
            bw.close();
        } catch (IOException e) {
            System.out.println(e.getMessage());
        }

OS X Framework Library not loaded: 'Image not found'

When you drag a custom framework into a project under Xcode 10.1, it assumes that the framework is a system framework and puts the framework into "Link Binary With Libraries" section of "Build Phases" under your target.

System frameworks are already on the device so it is not copied over to the device and thus cannot execute at runtime so KABOOM (crash in __abort_with_payload, and disinforming error: "Reason: image not found"). This is because the framework code is not copied to the device...

In reality, to have Xcode both link the custom framework and ensure that it is copied along with your code to the iOS device (real or simulator) the custom framework needs to be moved to "Copy Bundle Resources". This ultimately packages the framework along with your code executable to be available on the device together.

To add a custom framework to a project and avoid the Apple crash:

  1. Drag custom framework into your iOS project file list
  2. Click ProjectName in Navigator -> TargetName -> "Build Phases" -> Link Binary With Libraries disclosure triangle
  3. Drag custom framework out and down to "Copy Bundle Resources" section below (Xcode now moves the framework reference, fixed in Xcode 10)
  4. Run in simulator or device

The custom framework thus gets copied along with your code to your target device and is available at runtime.

enter image description here

[editorial: you would think Xcode would be smart enough to figure out the difference between one of it's system frameworks which need not be copied to the device and a custom framework that is, oh I don't know, in the project root directory hierarchy... ]

Unable to ping vmware guest from another vmware guest

  1. Try installing VMware tools in guest operating system.
  2. Check if firewall is enable
  3. If 1 and 2 are ok, try using share internet connection

Share internet

After sharing connection the VMnet8 IP address will be changed to 192.168.137.1, set up the IP 192.168.18.1 and try again

Search of table names

Adding on to @[RichardTheKiwi]'s answer.

Whenever I search for a list of tables, in general I want to select from all of them or delete them. Below is a script that generates those scripts for you.

The generated select script also adds a tableName column so you know what table you're looking at:

select 'select ''' + name + ''' as TableName, * from ' + name as SelectTable,
'delete from ' + name as DeleteTable
from sys.tables
where name like '%xxxx%'
and is_ms_shipped = 0; 

How to show all shared libraries used by executables in Linux?

Check shared library dependencies of a program executable

To find out what libraries a particular executable depends on, you can use ldd command. This command invokes dynamic linker to find out library dependencies of an executable.

> $ ldd /path/to/program

Note that it is NOT recommended to run ldd with any untrusted third-party executable because some versions of ldd may directly invoke the executable to identify its library dependencies, which can be security risk.

Instead, a safer way to show library dependencies of an unknown application binary is to use the following command.

$ objdump -p /path/to/program | grep NEEDED

for more info

oracle diff: how to compare two tables?

I used Oracle SQL developer to export the table/s into CSV format and then did the comparison using WinMerge.

Running sites on "localhost" is extremely slow

I know the op was using an older version of IIS and this may not apply to him, but I'm posting this as it might help others. I had the same problem and none of the above IPv6 or hosts file changes worked for me. My asp.net MVC4 project was really slow after hitting F5 to refresh js changes on localhost. It was happening across all browsers - Chrome, FF, and IE. Eventually I realised I was running IIS Express 8.0 locally, and it turns out 8.0 is extremely slow when serving up js files and seems to be a bug. If I ran iisexpress on the command line and hit F5 I could see each js file took 4 or 5 seconds to load.

I ended up uninstalling IIS 8.0 and installing IIS express 7.5 and straight away the problem was fixed. Here are the steps I followed:

IIS Express 8.0 seems to be installed with VS 2012 so if you had a new install or possibly a service pack update this might upgrade the previous IIS Express version.

Git command to display HEAD commit id?

You can use

git log -g branchname

to see git reflog information formatted like the git log output

TypeError: expected str, bytes or os.PathLike object, not _io.BufferedReader

I think it has to do with your second element in storbinary. You are trying to open file, but it is already a pointer to the file you opened in line file = open(local_path,'rb'). So, try to use ftp.storbinary("STOR " + i, file).

Comparing two byte arrays in .NET

 using System.Linq; //SequenceEqual

 byte[] ByteArray1 = null;
 byte[] ByteArray2 = null;

 ByteArray1 = MyFunct1();
 ByteArray2 = MyFunct2();

 if (ByteArray1.SequenceEqual<byte>(ByteArray2) == true)
 {
    MessageBox.Show("Match");
 }
 else
 {
   MessageBox.Show("Don't match");
 }

Command to collapse all sections of code?

If you mean shortcut then

CTRL + M + M: This one will collapse the region your cursor is at whether its a method, namespace or whatever for collapsing code blocks, regions and methods. The first will collapse only the block/method or region your cursor is at while the second will collapse the entire region you are at.

http://www.dev102.com/2008/05/06/11-more-visual-studio-shortcuts-you-should-know/

.htaccess not working apache

I cleared this use. By using this site click Here , follow the steps, the same steps follows upto the ubuntu version 18.04

How do I access store state in React Redux?

If you want to do some high-powered debugging, you can subscribe to every change of the state and pause the app to see what's going on in detail as follows.

store.js
store.subscribe( () => {
  console.log('state\n', store.getState());
  debugger;
});

Place that in the file where you do createStore.

To copy the state object from the console to the clipboard, follow these steps:

  1. Right-click an object in Chrome's console and select Store as Global Variable from the context menu. It will return something like temp1 as the variable name.

  2. Chrome also has a copy() method, so copy(temp1) in the console should copy that object to your clipboard.

https://stackoverflow.com/a/25140576

https://scottwhittaker.net/chrome-devtools/2016/02/29/chrome-devtools-copy-object.html

You can view the object in a json viewer like this one: http://jsonviewer.stack.hu/

You can compare two json objects here: http://www.jsondiff.com/

Emulator error: This AVD's configuration is missing a kernel file

For me Updating the SDK Tools fixed the errors.

Screenshot of the errors and update progress

how to convert .java file to a .class file

Use the javac program that comes with the JDK (visit java.sun.com if you don't have it). The installer does not automatically add javac to your system path, so you'll need to add the bin directory of the installed path to your system path before you can use it easily.

How to get my project path?

Your program has no knowledge of where your VS project is, so see get path for my .exe and go ../.. to get your project's path.

Doctrine query builder using inner join with conditions

I'm going to answer my own question.

  1. innerJoin should use the keyword "WITH" instead of "ON" (Doctrine's documentation [13.2.6. Helper methods] is inaccurate; [13.2.5. The Expr class] is correct)
  2. no need to link foreign keys in join condition as they're already specified in the entity mapping.

Therefore, the following works for me

$qb->select('c')
    ->innerJoin('c.phones', 'p', 'WITH', 'p.phone = :phone')
    ->where('c.username = :username');

or

$qb->select('c')
    ->innerJoin('c.phones', 'p', Join::WITH, $qb->expr()->eq('p.phone', ':phone'))
    ->where('c.username = :username');

Do we need to execute Commit statement after Update in SQL Server

The SQL Server Management Studio has implicit commit turned on, so all statements that are executed are implicitly commited.

This might be a scary thing if you come from an Oracle background where the default is to not have commands commited automatically, but it's not that much of a problem.

If you still want to use ad-hoc transactions, you can always execute

BEGIN TRANSACTION

within SSMS, and than the system waits for you to commit the data.

If you want to replicate the Oracle behaviour, and start an implicit transaction, whenever some DML/DDL is issued, you can set the SET IMPLICIT_TRANSACTIONS checkbox in

Tools -> Options -> Query Execution -> SQL Server -> ANSI

C++ String Concatenation operator<<

You can combine strings using stream string like that:

#include <iostream>
#include <sstream>
using namespace std;
int main()
{
    string name = "Bill";
    stringstream ss;
    ss << "Your name is: " << name;
    string info = ss.str();
    cout << info << endl;
    return 0;
}

How do I find files that do not contain a given string pattern?

You can do it with grep alone (without find).

grep -riL "foo" .

This is the explanation of the parameters used on grep

     -L, --files-without-match
             each file processed.
     -R, -r, --recursive
             Recursively search subdirectories listed.

     -i, --ignore-case
             Perform case insensitive matching.

If you use l (lowercased) you will get the opposite (files with matches)

     -l, --files-with-matches
             Only the names of files containing selected lines are written

NULL value for int in Update statement

Assuming the column is set to support NULL as a value:

UPDATE YOUR_TABLE
   SET column = NULL

Be aware of the database NULL handling - by default in SQL Server, NULL is an INT. So if the column is a different data type you need to CAST/CONVERT NULL to the proper data type:

UPDATE YOUR_TABLE
   SET column = CAST(NULL AS DATETIME)

...assuming column is a DATETIME data type in the example above.

Adding a library/JAR to an Eclipse Android project

Setting up a Library Project

A library project is a standard Android project, so you can create a new one in the same way as you would a new application project.

When you are creating the library project, you can select any application name, package, and set other fields as needed, as shown in figure 1.

Next, set the project's properties to indicate that it is a library project:

In the Package Explorer, right-click the library project and select Properties. In the Properties window, select the "Android" properties group at left and locate the Library properties at right. Select the "is Library" checkbox and click Apply. Click OK to close the Properties window. The new project is now marked as a library project. You can begin moving source code and resources into it, as described in the sections below.

How can I find an element by CSS class with XPath?

This selector should work but will be more efficient if you replace it with your suited markup:

//*[contains(@class, 'Test')]

Or, since we know the sought element is a div:

//div[contains(@class, 'Test')]

But since this will also match cases like class="Testvalue" or class="newTest", @Tomalak's version provided in the comments is better:

//div[contains(concat(' ', @class, ' '), ' Test ')]

If you wished to be really certain that it will match correctly, you could also use the normalize-space function to clean up stray whitespace characters around the class name (as mentioned by @Terry):

//div[contains(concat(' ', normalize-space(@class), ' '), ' Test ')]

Note that in all these versions, the * should best be replaced by whatever element name you actually wish to match, unless you wish to search each and every element in the document for the given condition.

What is the (function() { } )() construct in JavaScript?

It’s an Immediately-Invoked Function Expression, or IIFE for short. It executes immediately after it’s created.

It has nothing to do with any event-handler for any events (such as document.onload).
Consider the part within the first pair of parentheses: (function(){})();....it is a regular function expression. Then look at the last pair (function(){})();, this is normally added to an expression to call a function; in this case, our prior expression.

This pattern is often used when trying to avoid polluting the global namespace, because all the variables used inside the IIFE (like in any other normal function) are not visible outside its scope.
This is why, maybe, you confused this construction with an event-handler for window.onload, because it’s often used as this:

(function(){
  // all your code here
  var foo = function() {};
  window.onload = foo;
  // ...
})();
// foo is unreachable here (it’s undefined)

Correction suggested by Guffa:

The function is executed right after it's created, not after it is parsed. The entire script block is parsed before any code in it is executed. Also, parsing code doesn't automatically mean that it's executed, if for example the IIFE is inside a function then it won't be executed until the function is called.

Update Since this is a pretty popular topic, it's worth mentioning that IIFE's can also be written with ES6's arrow function (like Gajus has pointed out in a comment) :

((foo) => {
 // do something with foo here foo
})('foo value')

HTTP Error 403.14 - Forbidden - The Web server is configured to not list the contents of this directory

On the site in IIS:

  1. select 'Advance Settings'
  2. Then for application pool choose "ASP.NET v4.0"

image for example

Send File Attachment from Form Using phpMailer and PHP

You'd use $_FILES['uploaded_file']['tmp_name'], which is the path where PHP stored the uploaded file (it's a temporary file, removed automatically by PHP when the script ends, unless you've moved/copied it elsewhere).

Assuming your client-side form and server-side upload settings are correct, there's nothing you have to do to "pull in" the upload. It'll just magically be available in that tmp_name path.

Note that you WILL have to validate that the upload actually succeeded, e.g.

if ($_FILES['uploaded_file']['error'] === UPLOAD_ERR_OK) {
    ... attach file to email ...
}

Otherwise you may try to do an attachment with a damaged/partial/non-existent file.

Node / Express: EADDRINUSE, Address already in use - Kill server

You can also go the command line route:

ps aux | grep node

to get the process ids.

Then:

kill -9 PID

Doing the -9 on kill sends a SIGKILL (instead of a SIGTERM). SIGTERM has been ignored by node for me sometimes.

How to run .APK file on emulator

Steps (These apply for Linux. For other OS, visit here) -

  1. Copy the apk file to platform-tools in android-sdk linux folder.
  2. Open Terminal and navigate to platform-tools folder in android-sdk.
  3. Then Execute this command -

    ./adb install FileName.apk

  4. If the operation is successful (the result is displayed on the screen), then you will find your file in the launcher of your emulator.

For more info can check this link : android videos

When to use margin vs padding in CSS

Margin

Margin is usually used to create a space between the element itself and its surround.

for example I use it when I'm building a navbar to make it sticks to the edges of the screen and for no white gap.

Padding

I usually use when I've an element inside a border, <div> or something similar, and I want to decrease its size but at the time I want to keep the distance or the margin between the other elements around it.

So briefly, it's situational; it depends on what you are trying to do.

How to hide html source & disable right click and text copy?

They do this with some basic javascript, but this does not actually hide your HTML source! In many browsers you can simply go to view->source on the menu. Even if you couldn't, it is trivial to simply load up a debugging proxy like Fiddler, or packet-sniff the connection.

It is impossible to effectively hide the HTML, JavaScript, or any other resource sent to the client. Impossible, and isn't all that useful either.

Furthermore, don't try to disable right-click, as there are many other items on that menu (such as print!) that people use regularly.

Batch Extract path and filename from a variable

@ECHO OFF
SETLOCAL
set file=C:\Users\l72rugschiri\Desktop\fs.cfg
FOR %%i IN ("%file%") DO (
ECHO filedrive=%%~di
ECHO filepath=%%~pi
ECHO filename=%%~ni
ECHO fileextension=%%~xi
)

Not really sure what you mean by no "function"

Obviously, change ECHO to SET to set the variables rather thon ECHOing them...

See for documentation for a full list.


ceztko's test case (for reference)

@ECHO OFF
SETLOCAL
set file="C:\Users\ l72rugschiri\Desktop\fs.cfg"
FOR /F "delims=" %%i IN ("%file%") DO (
ECHO filedrive=%%~di
ECHO filepath=%%~pi
ECHO filename=%%~ni
ECHO fileextension=%%~xi
)

Comment : please see comments.

Less than or equal to

There is no => for if.
Use if %energy% GEQ %m2enc%

See if /? for some other details.

Android open pdf file

As of API 24, sending a file:// URI to another app will throw a FileUriExposedException. Instead, use FileProvider to send a content:// URI:

public File getFile(Context context, String fileName) {
    if (!Environment.getExternalStorageState().equals(Environment.MEDIA_MOUNTED)) {
        return null;
    }

    File storageDir = context.getExternalFilesDir(null);
    return new File(storageDir, fileName);
}

public Uri getFileUri(Context context, String fileName) {
    File file = getFile(context, fileName);
    return FileProvider.getUriForFile(context, BuildConfig.APPLICATION_ID + ".provider", file);
}

You must also define the FileProvider in your manifest:

<provider
    android:name="android.support.v4.content.FileProvider"
    android:authorities="com.mydomain.fileprovider"
    android:exported="false"
    android:grantUriPermissions="true">
    <meta-data
        android:name="android.support.FILE_PROVIDER_PATHS"
        android:resource="@xml/file_paths" />
</provider>

Example file_paths.xml:

<?xml version="1.0" encoding="utf-8"?>
<paths xmlns:android="http://schemas.android.com/apk/res/android">
    <external-files-path name="name" path="path" />
</paths>

Replace "name" and "path" as appropriate.

To give the PDF viewer access to the file, you also have to add the FLAG_GRANT_READ_URI_PERMISSION flag to the intent:

private void displayPdf(String fileName) {
    Uri uri = getFileUri(this, fileName);

    Intent intent = new Intent(Intent.ACTION_VIEW);
    intent.setDataAndType(uri, "application/pdf");

    // FLAG_GRANT_READ_URI_PERMISSION is needed on API 24+ so the activity opening the file can read it
    intent.setFlags(Intent.FLAG_ACTIVITY_NO_HISTORY | Intent.FLAG_GRANT_READ_URI_PERMISSION);

    if (intent.resolveActivity(getPackageManager()) == null) {
        // Show an error
    } else {
        startActivity(intent);
    }
}

See the FileProvider documentation for more details.

How to add Apache HTTP API (legacy) as compile-time dependency to build.grade for Android M?

For API 23:

Top level build.gradle - /build.gradle

buildscript {
    ...
    dependencies {
        classpath 'com.android.tools.build:gradle:1.3.1'
    }
}
...

Module specific build.gradle - /app/build.gradle

android {
    compileSdkVersion 23
    buildToolsVersion "23.0.0"
    useLibrary 'org.apache.http.legacy'
    ...
}

Official docs (for preview though): http://developer.android.com/about/versions/marshmallow/android-6.0-changes.html#behavior-apache-http-client

Latest android gradle plugin changelog: http://tools.android.com/tech-docs/new-build-system

Save each sheet in a workbook to separate CSV files

Here is one that will give you a visual file chooser to pick the folder you want to save the files to and also lets you choose the CSV delimiter (I use pipes '|' because my fields contain commas and I don't want to deal with quotes):

' ---------------------- Directory Choosing Helper Functions -----------------------
' Excel and VBA do not provide any convenient directory chooser or file chooser
' dialogs, but these functions will provide a reference to a system DLL
' with the necessary capabilities
Private Type BROWSEINFO    ' used by the function GetFolderName
    hOwner As Long
    pidlRoot As Long
    pszDisplayName As String
    lpszTitle As String
    ulFlags As Long
    lpfn As Long
    lParam As Long
    iImage As Long
End Type

Private Declare Function SHGetPathFromIDList Lib "shell32.dll" _
                                             Alias "SHGetPathFromIDListA" (ByVal pidl As Long, ByVal pszPath As String) As Long
Private Declare Function SHBrowseForFolder Lib "shell32.dll" _
                                           Alias "SHBrowseForFolderA" (lpBrowseInfo As BROWSEINFO) As Long

Function GetFolderName(Msg As String) As String
    ' returns the name of the folder selected by the user
    Dim bInfo As BROWSEINFO, path As String, r As Long
    Dim X As Long, pos As Integer
    bInfo.pidlRoot = 0&    ' Root folder = Desktop
    If IsMissing(Msg) Then
        bInfo.lpszTitle = "Select a folder."
        ' the dialog title
    Else
        bInfo.lpszTitle = Msg    ' the dialog title
    End If
    bInfo.ulFlags = &H1    ' Type of directory to return
    X = SHBrowseForFolder(bInfo)    ' display the dialog
    ' Parse the result
    path = Space$(512)
    r = SHGetPathFromIDList(ByVal X, ByVal path)
    If r Then
        pos = InStr(path, Chr$(0))
        GetFolderName = Left(path, pos - 1)
    Else
        GetFolderName = ""
    End If
End Function
'---------------------- END Directory Chooser Helper Functions ----------------------

Public Sub DoTheExport()
    Dim FName As Variant
    Dim Sep As String
    Dim wsSheet As Worksheet
    Dim nFileNum As Integer
    Dim csvPath As String


    Sep = InputBox("Enter a single delimiter character (e.g., comma or semi-colon)", _
                   "Export To Text File")
    'csvPath = InputBox("Enter the full path to export CSV files to: ")

    csvPath = GetFolderName("Choose the folder to export CSV files to:")
    If csvPath = "" Then
        MsgBox ("You didn't choose an export directory. Nothing will be exported.")
        Exit Sub
    End If

    For Each wsSheet In Worksheets
        wsSheet.Activate
        nFileNum = FreeFile
        Open csvPath & "\" & _
             wsSheet.Name & ".csv" For Output As #nFileNum
        ExportToTextFile CStr(nFileNum), Sep, False
        Close nFileNum
    Next wsSheet

End Sub



Public Sub ExportToTextFile(nFileNum As Integer, _
                            Sep As String, SelectionOnly As Boolean)

    Dim WholeLine As String
    Dim RowNdx As Long
    Dim ColNdx As Integer
    Dim StartRow As Long
    Dim EndRow As Long
    Dim StartCol As Integer
    Dim EndCol As Integer
    Dim CellValue As String

    Application.ScreenUpdating = False
    On Error GoTo EndMacro:

    If SelectionOnly = True Then
        With Selection
            StartRow = .Cells(1).Row
            StartCol = .Cells(1).Column
            EndRow = .Cells(.Cells.Count).Row
            EndCol = .Cells(.Cells.Count).Column
        End With
    Else
        With ActiveSheet.UsedRange
            StartRow = .Cells(1).Row
            StartCol = .Cells(1).Column
            EndRow = .Cells(.Cells.Count).Row
            EndCol = .Cells(.Cells.Count).Column
        End With
    End If

    For RowNdx = StartRow To EndRow
        WholeLine = ""
        For ColNdx = StartCol To EndCol
            If Cells(RowNdx, ColNdx).Value = "" Then
                CellValue = ""
            Else
                CellValue = Cells(RowNdx, ColNdx).Value
            End If
            WholeLine = WholeLine & CellValue & Sep
        Next ColNdx
        WholeLine = Left(WholeLine, Len(WholeLine) - Len(Sep))
        Print #nFileNum, WholeLine
    Next RowNdx

EndMacro:
    On Error GoTo 0
    Application.ScreenUpdating = True

End Sub

What is an MDF file?

Just to make this absolutely clear for all:

A .MDF file is “typically” a SQL Server data file however it is important to note that it does NOT have to be.

This is because .MDF is nothing more than a recommended/preferred notation but the extension itself does not actually dictate the file type.

To illustrate this, if someone wanted to create their primary data file with an extension of .gbn they could go ahead and do so without issue.

To qualify the preferred naming conventions:

  • .mdf - Primary database data file.
  • .ndf - Other database data files i.e. non Primary.
  • .ldf - Log data file.

Yii2 data provider default sorting

Try to this one

$dataProvider = new ActiveDataProvider([
    'query' => $query,
]);

$sort = $dataProvider->getSort();

$sort->defaultOrder = ['id' => SORT_ASC];

$dataProvider->setSort($sort);

YAML Multi-Line Arrays

If what you are needing is an array of arrays, you can do this way:

key:
  - [ 'value11', 'value12', 'value13' ]
  - [ 'value21', 'value22', 'value23' ]

Detect & Record Audio in Python

import pyaudio
import wave
from array import array

FORMAT=pyaudio.paInt16
CHANNELS=2
RATE=44100
CHUNK=1024
RECORD_SECONDS=15
FILE_NAME="RECORDING.wav"

audio=pyaudio.PyAudio() #instantiate the pyaudio

#recording prerequisites
stream=audio.open(format=FORMAT,channels=CHANNELS, 
                  rate=RATE,
                  input=True,
                  frames_per_buffer=CHUNK)

#starting recording
frames=[]

for i in range(0,int(RATE/CHUNK*RECORD_SECONDS)):
    data=stream.read(CHUNK)
    data_chunk=array('h',data)
    vol=max(data_chunk)
    if(vol>=500):
        print("something said")
        frames.append(data)
    else:
        print("nothing")
    print("\n")


#end of recording
stream.stop_stream()
stream.close()
audio.terminate()
#writing to file
wavfile=wave.open(FILE_NAME,'wb')
wavfile.setnchannels(CHANNELS)
wavfile.setsampwidth(audio.get_sample_size(FORMAT))
wavfile.setframerate(RATE)
wavfile.writeframes(b''.join(frames))#append frames recorded to file
wavfile.close()

I think this will help.It is a simple script which will check if there is a silence or not.If silence is detected it will not record otherwise it will record.

Passing a local variable from one function to another

Adding to @pranay-rana's list:

Third way is:

function passFromValue(){
    var x = 15;
    return x;  
}
function passToValue() {
    var y = passFromValue();
    console.log(y);//15
}
passToValue(); 

Options for HTML scraping?

Scraping Stack Overflow is especially easy with Shoes and Hpricot.

require 'hpricot'

Shoes.app :title => "Ask Stack Overflow", :width => 370 do
  SO_URL = "http://stackoverflow.com"
  stack do
    stack do
      caption "What is your question?"
      flow do
        @lookup = edit_line "stackoverflow", :width => "-115px"
        button "Ask", :width => "90px" do
          download SO_URL + "/search?s=" + @lookup.text do |s|
            doc = Hpricot(s.response.body)
            @rez.clear()
            (doc/:a).each do |l|
              href = l["href"]
              if href.to_s =~ /\/questions\/[0-9]+/ then
                @rez.append do
                  para(link(l.inner_text) { visit(SO_URL + href) })
                end
              end
            end
            @rez.show()
          end
        end
      end
    end
    stack :margin => 25 do
      background white, :radius => 20
      @rez = stack do
      end
    end
    @rez.hide()
  end
end

How do I compare two strings in Perl?

And if you'd like to extract the differences between the two strings, you can use String::Diff.

Create multiple threads and wait all of them to complete

If you don't want to use the Task class (for instance, in .NET 3.5) you can just start all your threads, and then add them to the list and join them in a foreach loop.

Example:

List<Thread> threads = new List<Thread>();


// Start threads
for(int i = 0; i<10; i++)
{
    int tmp = i; // Copy value for closure
    Thread t = new Thread(() => Console.WriteLine(tmp));
    t.Start;
    threads.Add(t);
}

// Await threads
foreach(Thread thread in threads)
{
    thread.Join();
}

Set a default parameter value for a JavaScript function

As per the syntax

function [name]([param1[ = defaultValue1 ][, ..., paramN[ = defaultValueN ]]]) {
   statements
}

you can define the default value of formal parameters. and also check undefined value by using typeof function.

pod install -bash: pod: command not found

  1. Uninstall all instances of cocopods by this command

    $sudo gem uninstall cocoapods
    
  2. sudo gem install -n /usr/local/bin cocoapods

  3. sudo chmod +rx /usr/local/bin/

numpy: most efficient frequency counts for unique values in an array

As of Numpy 1.9, the easiest and fastest method is to simply use numpy.unique, which now has a return_counts keyword argument:

import numpy as np

x = np.array([1,1,1,2,2,2,5,25,1,1])
unique, counts = np.unique(x, return_counts=True)

print np.asarray((unique, counts)).T

Which gives:

 [[ 1  5]
  [ 2  3]
  [ 5  1]
  [25  1]]

A quick comparison with scipy.stats.itemfreq:

In [4]: x = np.random.random_integers(0,100,1e6)

In [5]: %timeit unique, counts = np.unique(x, return_counts=True)
10 loops, best of 3: 31.5 ms per loop

In [6]: %timeit scipy.stats.itemfreq(x)
10 loops, best of 3: 170 ms per loop

Is there a Social Security Number reserved for testing/examples?

To expand on the Wikipedia-based answers:

The Social Security Administration (SSA) explicitly states in this document that the having "000" in the first group of numbers "will NEVER be a valid SSN":

I'd consider that pretty definitive.

However, that the 2nd or 3rd groups of numbers won't be "00" or "0000" can be inferred from a FAQ that the SSA publishes which indicates that allocation of those groups starts at "01" or "0001":

But this is only a FAQ and it's never outright stated that "00" or "0000" will never be used.

In another FAQ they provide (http://www.socialsecurity.gov/employer/randomizationfaqs.html#a0=6) that "00" or "0000" will never be used.

I can't find a reference to the 'advertisement' reserved SSNs on the SSA site, but it appears that no numbers starting with a 3 digit number higher than 772 (according to the document referenced above) have been assigned yet, but there's nothing I could find that states those numbers are reserved. Wikipedia's reference is a book that I don't have access to. The Wikipedia information on the advertisement reserved numbers is mentioned across the web, but many are clearly copied from Wikipedia. I think it would be nice to have a citation from the SSA, though I suspect that now that Wikipedia has made the idea popular that these number would now have to be reserved for advertisements even if they weren't initially.

The SSA has a page with a couple of stories about SSN's they've had to retire because they were used in advertisements/samples (maybe the SSA should post a link to whatever their current policy on this might be):

How to save a figure in MATLAB from the command line?

try plot(var); saveFigure('title'); it will save as a jpeg automatically

Spring Boot REST service exception handling

With Spring Boot 1.4+ new cool classes for easier exception handling were added that helps in removing the boilerplate code.

A new @RestControllerAdvice is provided for exception handling, it is combination of @ControllerAdvice and @ResponseBody. You can remove the @ResponseBody on the @ExceptionHandler method when use this new annotation.

i.e.

@RestControllerAdvice
public class GlobalControllerExceptionHandler {

    @ExceptionHandler(value = { Exception.class })
    @ResponseStatus(HttpStatus.INTERNAL_SERVER_ERROR)
    public ApiErrorResponse unknownException(Exception ex, WebRequest req) {
        return new ApiErrorResponse(...);
    }
}

For handling 404 errors adding @EnableWebMvc annotation and the following to application.properties was enough:
spring.mvc.throw-exception-if-no-handler-found=true

You can find and play with the sources here:
https://github.com/magiccrafter/spring-boot-exception-handling

The type or namespace name does not exist in the namespace 'System.Web.Mvc'

I have deleted System.Web.dll from Bin frolder of my site.

android View not attached to window manager

After a fight with this issue, I finally end up with this workaround:

/**
 * Dismiss {@link ProgressDialog} with check for nullability and SDK version
 *
 * @param dialog instance of {@link ProgressDialog} to dismiss
 */
public void dismissProgressDialog(ProgressDialog dialog) {
    if (dialog != null && dialog.isShowing()) {

            //get the Context object that was used to great the dialog
            Context context = ((ContextWrapper) dialog.getContext()).getBaseContext();

            // if the Context used here was an activity AND it hasn't been finished or destroyed
            // then dismiss it
            if (context instanceof Activity) {

                // Api >=17
                if (!((Activity) context).isFinishing() {
                    if (Build.VERSION.SDK_INT >= Build.VERSION_CODES.JELLY_BEAN_MR1) {
                        if (!((Activity) context).isDestroyed()) {
                            dismissWithExceptionHandling(dialog);
                        }
                    } else { 
                        // Api < 17. Unfortunately cannot check for isDestroyed()
                        dismissWithExceptionHandling(dialog);
                    }
                }
            } else
                // if the Context used wasn't an Activity, then dismiss it too
                dismissWithExceptionHandling(dialog);
        }
        dialog = null;
    }
}

/**
 * Dismiss {@link ProgressDialog} with try catch
 *
 * @param dialog instance of {@link ProgressDialog} to dismiss
 */
public void dismissWithExceptionHandling(ProgressDialog dialog) {
    try {
        dialog.dismiss();
    } catch (final IllegalArgumentException e) {
        // Do nothing.
    } catch (final Exception e) {
        // Do nothing.
    } finally {
        dialog = null;
    }
}

Sometimes, good exception handling works well if there wasn't a better solution for this issue.

Setting java locale settings

If you are on Mac, simply using System Preferences -> Languages and dragging the language to test to top (before English) will make sure the next time you open the App, the right locale is tried!!

no overload for matches delegate 'system.eventhandler'

Change the klik method as follows:

public void klik(object pea, EventArgs e)
{
    Bitmap c = this.DrawMandel();
    Button btn = pea as Button;
    Graphics gr = btn.CreateGraphics();
    gr.DrawImage(b, 150, 200);
}

Create an array of integers property in Objective-C

I found all the previous answers too much complicated. I had the need to store an array of some ints as a property, and found the ObjC requirement of using a NSArray an unneeded complication of my software.

So I used this:

typedef struct my10ints {
    int arr[10];
} my10ints;

@interface myClasss : NSObject

@property my10ints doubleDigits;

@end

This compiles cleanly using Xcode 6.2.

My intention was to use it like this:

myClass obj;
obj.doubleDigits.arr[0] = 4;

HOWEVER, this does not work. This is what it produces:

int i = 4;
myClass obj;
obj.doubleDigits.arr[0] = i;
i = obj.doubleDigits.arr[0];
// i is now 0 !!!

The only way to use this correctly is:

int i = 4;
myClass obj;
my10ints ints;
ints = obj.doubleDigits;
ints.arr[0] = i;
obj.doubleDigits = ints;
i = obj.doubleDigits.arr[0];
// i is now 4

and so, defeats completely my point (avoiding the complication of using a NSArray).

subquery in codeigniter active record

For query: SELECT * FROM (SELECT id, product FROM product) as product you can use:

$sub_query_from = '(SELECT id, product FROM product ) as product';
$this->db->select();
$this->db->from($sub_query_from);
$query = $this->db->get()

Please notice, that in sub_query_from string you must use spaces between ... product ) as...

How to create a project from existing source in Eclipse and then find it?

This answer is going to be for the question

How to create a new eclipse project and add a folder or a new package into the project, or how to build a new project for existing java files.

  1. Create a new project from the menu File->New-> Java Project
  2. If you are going to add a new pakcage, then create the same package name here by File->New-> Package
  3. Click the name of the package in project navigator, and right click, and import... Import->General->File system (choose your file or package)

this worked for me I hope it helps others. Thank you.

Resolve host name to an ip address

Go to your client machine and type in:

nslookup server.company.com

substituting the real host name of your server for server.company.com, of course.

That should tell you which DNS server your client is using (if any) and what it thinks the problem is with the name.

To force an application to use an IP address, generally you just configure it to use the IP address instead of a host name. If the host name is hard-coded, or the application insists on using a host name in preference to an IP address (as one of your other comments seems to indicate), then you're probably out of luck there.

However, you can change the way that most machine resolve the host names, such as with /etc/resolv.conf and /etc/hosts on UNIXy systems and a local hosts file on Windows-y systems.

C++ printing spaces or tabs given a user input integer

Appending single space to output file with stream variable.

// declare output file stream varaible and open file ofstream fout; fout.open("flux_capacitor.txt"); fout << var << " ";

Android turn On/Off WiFi HotSpot programmatically

WifiManager wifiManager = (WifiManager)this.context.getSystemService(Context.WIFI_SERVICE);
wifiManager.setWifiEnabled(status);

where status may be true or false

add permission manifest: <uses-permission android:name="android.permission.CHANGE_WIFI_STATE" />

Node.js Port 3000 already in use but it actually isn't?

It may be an admin process running in the background and netstat doesn't show this.
Use tasklist | grep node to find the PID of this admin process and then kill PID

PHP Fatal error: Uncaught exception 'Exception'

For

throw new Exception('test exception');

I got 500 (but didn't see anything in the browser), until I put

php_flag display_errors on

in my .htaccess (just for a subfolder). There are also more detailed settings, see Enabling error display in php via htaccess only

foreach vs someList.ForEach(){}

We had some code here (in VS2005 and C#2.0) where the previous engineers went out of their way to use list.ForEach( delegate(item) { foo;}); instead of foreach(item in list) {foo; }; for all the code that they wrote. e.g. a block of code for reading rows from a dataReader.

I still don't know exactly why they did this.

The drawbacks of list.ForEach() are:

  • It is more verbose in C# 2.0. However, in C# 3 onwards, you can use the "=>" syntax to make some nicely terse expressions.

  • It is less familiar. People who have to maintain this code will wonder why you did it that way. It took me awhile to decide that there wasn't any reason, except maybe to make the writer seem clever (the quality of the rest of the code undermined that). It was also less readable, with the "})" at the end of the delegate code block.

  • See also Bill Wagner's book "Effective C#: 50 Specific Ways to Improve Your C#" where he talks about why foreach is preferred to other loops like for or while loops - the main point is that you are letting the compiler decide the best way to construct the loop. If a future version of the compiler manages to use a faster technique, then you will get this for free by using foreach and rebuilding, rather than changing your code.

  • a foreach(item in list) construct allows you to use break or continue if you need to exit the iteration or the loop. But you cannot alter the list inside a foreach loop.

I'm surprised to see that list.ForEach is slightly faster. But that's probably not a valid reason to use it throughout , that would be premature optimisation. If your application uses a database or web service that, not loop control, is almost always going to be be where the time goes. And have you benchmarked it against a for loop too? The list.ForEach could be faster due to using that internally and a for loop without the wrapper would be even faster.

I disagree that the list.ForEach(delegate) version is "more functional" in any significant way. It does pass a function to a function, but there's no big difference in the outcome or program organisation.

I don't think that foreach(item in list) "says exactly how you want it done" - a for(int 1 = 0; i < count; i++) loop does that, a foreach loop leaves the choice of control up to the compiler.

My feeling is, on a new project, to use foreach(item in list) for most loops in order to adhere to the common usage and for readability, and use list.Foreach() only for short blocks, when you can do something more elegantly or compactly with the C# 3 "=>" operator. In cases like that, there may already be a LINQ extension method that is more specific than ForEach(). See if Where(), Select(), Any(), All(), Max() or one of the many other LINQ methods doesn't already do what you want from the loop.

how to check confirm password field in form without reloading page

If you don't want use jQuery:

function check_pass() {
    if (document.getElementById('password').value ==
            document.getElementById('confirm_password').value) {
        document.getElementById('submit').disabled = false;
    } else {
        document.getElementById('submit').disabled = true;
    }
}
<input type="password" name="password" id="password" onchange='check_pass();'/>
<input type="password" name="confirm_password" id="confirm_password" onchange='check_pass();'/>
<input type="submit" name="submit"  value="registration"  id="submit" disabled/>

Managing SSH keys within Jenkins for Git

It looks like the github.com host which jenkins tries to connect to is not listed under the Jenkins user's $HOME/.ssh/known_hosts. Jenkins runs on most distros as the user jenkins and hence has its own .ssh directory to store the list of public keys and known_hosts.

The easiest solution I can think of to fix this problem is:

# Login as the jenkins user and specify shell explicity,
# since the default shell is /bin/false for most
# jenkins installations.
sudo su jenkins -s /bin/bash

cd SOME_TMP_DIR
# git clone YOUR_GITHUB_URL

# Allow adding the SSH host key to your known_hosts

# Exit from su
exit

Getting the error "Java.lang.IllegalStateException Activity has been destroyed" when using tabs with ViewPager

I'm having exactly the same problem. The only workaround I've found, is to replace the fragments by a new instance, each time the tabs are changed.

ft.replace(R.id.fragment_container, Fragment.instantiate(PlayerMainActivity.this, fragment.getClass().getName()));

Not a real solution, but i haven't found a way to reuse the previous fragment instance...

How to increase scrollback buffer size in tmux?

The history limit is a pane attribute that is fixed at the time of pane creation and cannot be changed for existing panes. The value is taken from the history-limit session option (the default value is 2000).

To create a pane with a different value you will need to set the appropriate history-limit option before creating the pane.

To establish a different default, you can put a line like the following in your .tmux.conf file:

set-option -g history-limit 3000

Note: Be careful setting a very large default value, it can easily consume lots of RAM if you create many panes.

For a new pane (or the initial pane in a new window) in an existing session, you can set that session’s history-limit. You might use a command like this (from a shell):

tmux set-option history-limit 5000 \; new-window

For (the initial pane of the initial window in) a new session you will need to set the “global” history-limit before creating the session:

tmux set-option -g history-limit 5000 \; new-session

Note: If you do not re-set the history-limit value, then the new value will be also used for other panes/windows/sessions created in the future; there is currently no direct way to create a single new pane/window/session with its own specific limit without (at least temporarily) changing history-limit (though show-option (especially in 1.7 and later) can help with retrieving the current value so that you restore it later).

How to hide the soft keyboard from inside a fragment?

Kotlin code

val imm = requireActivity().getSystemService(Context.INPUT_METHOD_SERVICE) as InputMethodManager
imm.hideSoftInputFromWindow(requireActivity().currentFocus?.windowToken, 0)

How to capitalize the first letter in a String in Ruby

Unfortunately, it is impossible for a machine to upcase/downcase/capitalize properly. It needs way too much contextual information for a computer to understand.

That's why Ruby's String class only supports capitalization for ASCII characters, because there it's at least somewhat well-defined.

What do I mean by "contextual information"?

For example, to capitalize i properly, you need to know which language the text is in. English, for example, has only two is: capital I without a dot and small i with a dot. But Turkish has four is: capital I without a dot, capital I with a dot, small i without a dot, small i with a dot. So, in English 'i'.upcase # => 'I' and in Turkish 'i'.upcase # => 'I'. In other words: since 'i'.upcase can return two different results, depending on the language, it is obviously impossible to correctly capitalize a word without knowing its language.

But Ruby doesn't know the language, it only knows the encoding. Therefore it is impossible to properly capitalize a string with Ruby's built-in functionality.

It gets worse: even with knowing the language, it is sometimes impossible to do capitalization properly. For example, in German, 'Maße'.upcase # => 'MASSE' (Maße is the plural of Maß meaning measurement). However, 'Masse'.upcase # => 'MASSE' (meaning mass). So, what is 'MASSE'.capitalize? In other words: correctly capitalizing requires a full-blown Artificial Intelligence.

So, instead of sometimes giving the wrong answer, Ruby chooses to sometimes give no answer at all, which is why non-ASCII characters simply get ignored in downcase/upcase/capitalize operations. (Which of course also reads to wrong results, but at least it's easy to check.)

How to quickly check if folder is empty (.NET)?

Some time you might want to verify whether any files exist inside sub directories and ignore those empty sub directories; in this case you can used method below:

public bool isDirectoryContainFiles(string path) {
    if (!Directory.Exists(path)) return false;
    return Directory.EnumerateFiles(path, "*", SearchOption.AllDirectories).Any();
}

In Python, how do I convert all of the items in a list to floats?

You can use numpy to convert a list directly to a floating array or matrix.

    import numpy as np
    list_ex = [1, 0] # This a list
    list_int = np.array(list_ex) # This is a numpy integer array

If you want to convert the integer array to a floating array then add 0. to it

    list_float = np.array(list_ex) + 0. # This is a numpy floating array

Maven Jacoco Configuration - Exclude classes/packages from report not working

Your XML is slightly wrong, you need to add any class exclusions within an excludes parent field, so your above configuration should look like the following as per the Jacoco docs

<configuration>
    <excludes>
        <exclude>**/*Config.*</exclude>
        <exclude>**/*Dev.*</exclude>
    </excludes>
</configuration>

The values of the exclude fields should be class paths (not package names) of the compiled classes relative to the directory target/classes/ using the standard wildcard syntax

*   Match zero or more characters
**  Match zero or more directories
?   Match a single character

You may also exclude a package and all of its children/subpackages this way:

<exclude>some/package/**/*</exclude>

This will exclude every class in some.package, as well as any children. For example, some.package.child wouldn't be included in the reports either.

I have tested and my report goal reports on a reduced number of classes using the above.

If you are then pushing this report into Sonar, you will then need to tell Sonar to exclude these classes in the display which can be done in the Sonar settings

Settings > General Settings > Exclusions > Code Coverage

Sonar Docs explains it a bit more

Running your command above

mvn clean verify

Will show the classes have been excluded

No exclusions

[INFO] --- jacoco-maven-plugin:0.7.4.201502262128:report (post-test) @ ** ---
[INFO] Analyzed bundle '**' with 37 classes

With exclusions

[INFO] --- jacoco-maven-plugin:0.7.4.201502262128:report (post-test) @ ** ---
[INFO] Analyzed bundle '**' with 34 classes

Hope this helps

make a phone call click on a button

I hope, this short code is useful for You,
   ## Java Code ##
 startActivity(new Intent(Intent.ACTION_DIAL,Uri.parse("tel:"+txtPhn.getText().toString())));



----------------------------------------------------------------------


Please check Manifest File,(for Uses permission)
## Manifest.xml ##
<manifest
    xmlns:android="http://schemas.android.com/apk/res/android"
    package="com.dbm.pkg"
    android:versionCode="1"
    android:versionName="1.0">

    <!-- NOTE! Your uses-permission must be outside the "application" tag
               but within the "manifest" tag. -->
## uses-permission for Making Call ##
    <uses-permission android:name="android.permission.CALL_PHONE" />

    <application
        android:icon="@drawable/icon"
        android:label="@string/app_name">

        <!-- Insert your other stuff here -->

    </application>

    <uses-sdk android:minSdkVersion="9" />
</manifest> 

How to specify jackson to only use fields - preferably globally

for jackson 1.9.10 I use

ObjectMapper mapper = new ObjectMapper();

mapper.setVisibility(JsonMethod.ALL, Visibility.NONE);
mapper.setVisibility(JsonMethod.FIELD, Visibility.ANY);

to turn of auto dedection.

How to suppress Pandas Future warning ?

Warnings are annoying. As mentioned in other answers, you can suppress them using:

import warnings
warnings.simplefilter(action='ignore', category=FutureWarning)

But if you want to handle them one by one and you are managing a bigger codebase, it will be difficult to find the line of code which is causing the warning. Since warnings unlike errors don't come with code traceback. In order to trace warnings like errors, you can write this at the top of the code:

import warnings
warnings.filterwarnings("error")

But if the codebase is bigger and it is importing bunch of other libraries/packages, then all sort of warnings will start to be raised as errors. In order to raise only certain type of warnings (in your case, its FutureWarning) as error, you can write:

import warnings
warnings.simplefilter(action='error', category=FutureWarning)

Why can I not create a wheel in python?

I also ran into the error message invalid command 'bdist_wheel'

It turns out the package setup.py used distutils rather than setuptools. Changing it as follows enabled me to build the wheel.

#from distutils.core import setup
from setuptools import setup